SC

Ace your homework & exams now with Quizwiz!

if would在一起处理

if would必然错

35. (Book Question: 801)Whereas in mammals the tiny tubes that convey nutrients to bone cells are arrayed in parallel lines, in birds the tubes form a random pattern. A. Whereas in mammals the tiny tubes that convey nutrients to bone cells are arrayed in parallel lines, in birds the tubes B. Whereas the tiny tubes for the conveying of nutrients to bone cells are arrayed in mammals in parallel lines, birds have tubes that C. Unlike mammals, where the tiny tubes for conveying nutrients to bone cells are arrayed in parallel lines, birds' tubes D. Unlike mammals, in whom the tiny tubes that convey nutrients to bone cells are arrayed in parallel lines, the tubes in birds E. Unlike the tiny tubes that convey nutrients to bone cells, which in mammals are arrayed in parallel lines, in birds the tubes

Subordinating conjunction: although, because, while, whereas Correct Answer: A Selected Answer: A Idiom; Rhetorical construction; Parallelism Whereas introduces two contrasting situations or events and should be followed by parallel structures. In this sentence, whereas is immediately followed by a clause beginning with the prepositional phrase in mammals; this means that the second part of the sentence must also be a clause that opens with a preposition that functions in the same way—in this case, in birds. This structure clarifies that the things being contrasted are the tubes in mammals and the tubes in birds. Incorrect versions of the sentence grammatically contrast tubes and birds, mammals and tubes, or birds and mammals. A Correct. Parallel structures make clear that the tubes in mammals are being contrasted with the tubes in birds. B The faulty parallelism results in a sentence that is confusing and unnecessarily wordy. C The sentence compares mammals and birds' tubes. D Because of faulty parallelism, this sentence also compares mammals and tubes in birds. E This structure is wordy and confusing because of faulty parallelism.

neither or either or be动词原则

When two subjects are joined by neither-nor or either-or, choosing the right number for the verb can be tricky for writers. Focus your attention on the noun closest to the verb. If it is singular, as in the sentence above, choose the singular verb. If the noun is plural, choose the plural form of the verb

136. (Book Question: 702)Traffic safety officials predict that drivers will be equally likely to exceed the proposed speed limit as the current one. A. equally likely to exceed the proposed speed limit as B. equally likely to exceed the proposed speed limit as they are C. equally likely that they will exceed the proposed speed limit as D. as likely that they will exceed the proposed speed limit as E. as likely to exceed the proposed speed limit as they are

Whenever you see "As" being used for comparison, it must be followed by a clause. This leaves us with two options B & E. Between B & E, E has the correct idiom usage as X as. Correct Answer: E Selected Answer: A Idiom; Parallelism This sentence reports on a prediction that compares the likelihood of drivers exceeding a proposed new speed limit with the likelihood of drivers exceeding the current speed limit. The idiom as x as y, rather than the incorrect form equally . . . as, should be used to express the comparison. A Equally likely . . . as is not an idiomatic form of comparison. B This also offers a nonidiomatic form of comparison. C The comparison is expressed nonidiomatically. Also, the drivers will be equally likely should be followed by to exceed rather than by that they will exceed. The resulting sentence is wordy and structurally flawed. D The resulting sentence is wordy and structurally flawed. The idiomatic phrase as x as y is somewhat in use, but as likely that they is awkward, and the comparison is unclear and not parallel. E Correct. The idiomatic phrase as x as y is properly used, and the comparison is clear and parallel.

15. (Book Question: 706)The Organization of Petroleum Exporting Countries (OPEC) had long been expected to announce a reduction in output to bolster sagging oil prices, but officials of the organization just recently announced that the group will pare daily production by 1.5 million barrels by the beginning of next year, but only if non-OPEC nations, including Norway, Mexico, and Russia, were to trim output by a total of 500,000 barrels a day. A. year, but only if non-OPEC nations, including Norway, Mexico, and Russia, were to trim output B. year, but only if the output of non-OPEC nations, which includes Norway, Mexico, and Russia, is trimmed C. year only if the output of non-OPEC nations, including Norway, Mexico, and Russia, would be trimmed D. year only if non-OPEC nations, which includes Norway, Mexico, and Russia, were trimming output E. year only if non-OPEC nations, including Norway, Mexico, and Russia, trim output

X will do something only if Y does something else. Correct Answer: E Selected Answer: E Rhetorical construction; Logical predication The underlined part of this sentence deals with the conditions under which OPEC members will lower their own oil production by 1.5 million barrels by the beginning of next year. The important thing to notice here is the following logical relation: X will do something only if Y does something else. A This version has redundant words, were to in front of trim that do not add anything more in meaning. Furthermore, were to trim is not the proper verb form to accompany will pare. The addition of but before only is also redundant. B This version uses the passive construction in the conditional clause only if the output . . . is trimmed. This use of the passive voice makes this sentence vague; it is now unclear who needs to trim the output of non-OPEC nations. Finally, the addition of but before only is redundant. C As in (B), this version also introduces vagueness by using the passive construction. In addition, would in front of the passive verb be trimmed is redundant. D This version uses an active verb, but in the past progressive form, were trimming. The progressive tense denotes actions in progress, so its use is not normally warranted in conditional sentences such as this one. E Correct. This version uses the correct and most concise conditional structure, without redundancies.

Many financial experts believe that policy makers at the Federal Reserve, now viewing the economy as balanced between moderate growth and low inflation, are almost certain to leave interests rates unchanged for the foreseeable future. A. Reserve, now viewing the economy as balanced between moderate growth and low inflation, are B. Reserve, now viewing the economy to be balanced between that of moderate growth and low inflation and are C. Reserve who, now viewing the economy as balanced between moderate growth and low inflation, are D. Reserve, who now view the economy to be balanced between that of moderate growth and low inflation, will be E. Reserve, which now views the economy to be balanced between moderate growth and low inflation, is

Yes you are correct, 'that of' is wrong here. It doesn't have anything to link back to. As others have said it is correct as in A

对"修饰"的汇总总结

[分享]对"修饰"的汇总总结 修饰 -------- 1.限定与非限定: 不加逗号的定从/with/-ing/-ed是限定的,大部分就近修饰;加逗号的定从/with/-ing/-ed是非限定的,可以就近修饰,也可以根据逻辑意思来修饰主句或从句的主语/谓语。不论限定还是非限定,都必须注意逻辑主语问题。 限定:"整体中的局部"概念,对被修饰成分下定义。 非限定:只表示这一个整体的特点,只提供被修饰成分的相关信息。OG10-258,OG11-49 ------- 2.介词结构: A.修饰主语以外的介词结构必须就近修饰。(OG11-78=OG10-113, with跳跃修饰monkey?因为相对好于whose定从?——定从必须就近修饰) B.当把with放在句末做修饰的时候要很小心, 因为with既可以就近修饰, 又可以修饰主句的主语(表伴随),所以会有歧义。若修饰主语,应将介词结构应该提到句首, 不能插在主谓之间(除非这个介词结构被封闭在从句或者同位语里面, 但绝对不能是单单的一个介词结构插在主谓之间OG32, 反例205???)。介词结构放句首时修饰全句,因此要注意是否有逻辑错误(OG133A 不能修饰and后半句所以错)。 C.with所修饰的名词就是它的逻辑宾语, 后面不可以再补出宾语(N, with ... doing it ) -------- 3.现在分词: 甲、 ","+现在分词短语 A.做插入语,位于句中就近修饰名词,是非限定性修饰语。如OG258B。 B.(1)放在一个句子(包括从句)的句末,而修饰前面句子的主语,表伴随动作/状态(如OG256/120)此时ing分词的逻辑主语一定要与主语一致。这种情况等价于-ing位于句首。 (2)放在一个句子(包括从句)的句末,前面整个句子作为一个整体导致后面的分词短语,表伴随结果 OG253 注意:一般的状语从句(时间/条件/原因从句)后跟ING分词会有歧义:ing是修饰从句的主语还是主句的主语?OG35 乙、"n.+现在分词短语": 就近修饰名词,做限定性修饰语 -------- 4.过去分词: 过去分词位于句首, 逻辑主语必须一致; 过去分词在句末(无论前面有没有逗号), 大多是就近修饰, 逻辑主语可以不一致。 -------- 5.分词、定语从句、介词结构的比较: A. 首先,一般情况下,定从、分词短语、介词结构之间没有优先级,三者可以互相转化;主要看逻辑主语是否正确,是否有修饰歧义,句子其它部分是否出错如主谓/指代等等。 B. 当涉及到完成时态时,分词短语必须转化为定语从句:当出现having done时,必须改为which have done的定语从句。 C. 如果其它部分没有错误,修饰也没有歧义,单纯比较"N, which"与"N, -ing"或"N which"与"N -ing",则前者的修饰要比后者的更加清晰 OG253 DE D. 定语从句的引导词(that/which/who/...)在从句中充当一定的成分;with短语/现在分词短语的引导词(with/in/on/...)不作成分 E. (1) , -ing常考修饰歧义或伴随: 修饰歧义:就近指代是否有逻辑错误 伴随:, -ing可以修饰前面的对象,也可以修饰紧跟的名词,存在歧义;改成:把分词放在句首,或独立主格 n+ -ing (2) , with常考伴随: 伴随: , with...在句末表伴随的选项大部分情况下都错;改成:with要么句首表伴随,要么独立主格 , with n... (3) , which常考修饰歧义或指代前句整句: 修饰歧义:就近修饰是否有逻辑错误(注意which不一定非得修饰","前的第一个名词:OG 114 ... the Menomini crafted a canoe about twenty feet long and two feet wide, with small ribs and rails of cedar, which could carry... 两个介词结构隔开宾语和which,which修饰最近的宾语) 指代前句整句:, WHICH不能指代前句整句,改成such/so, 或者, -ing修饰前句中的主语,表示伴随结果/动作。

According to a survey of graduating medical students conducted by Association of American Medical College, minority graduates are nearly four times more likely than are other graduates in planning to practice in socioeconomically deprived areas. (A) minority graduates are nearly four times more likely than are other graduates in planning to practice (B) minority graduates are nearly four times more likely than other graduates who plan on practicing (C) minority graduates are nearly four times as likely as are other graduates to plan on practicing (D) it is nearly four times more likely that minority graduates rather than other graduates will plan to practice (E) it is nearly four times as likely for minority graduates than other graduates to plan to practice

(A) minority graduates are nearly four times more likely than are other graduates in planning to practice Incorrect: Check: Minority Graduate Vs Other Graduates in Planning ( are they working in Planning Dept !!) (B) minority graduates are nearly four times more likely than other graduates who plan on practicing Incorrect: Here the Relative pronoun Who takes the Socioeco....part of sentence with Other Graduates. We are left with nothing to compare !! (C) minority graduates are nearly four times as likely as are other graduates to plan on practicing Correct: Here we mean to say that Minority Grads are 4 times likely to choose(plan) Socio..than other Grads. This comparision is perfect. Isnt it ? (D) it is nearly four times more likely that minority graduates rather than other graduates will plan to practice Incorrect: "WILL" Going tooo far man ? (E) it is nearly four times as likely for minority graduates than other graduates to plan to practice Incorrect: Do you mean to say : That the plan is 4 times more preferable to Min Grads than Other Grads !!

413. In the mid-1920's the Hawthorne Works of the Western Electric Company was the scene of an intensive series of experiments that would investigate changes in working conditions as to their effects on workers' performance. (A) that would investigate changes in working conditions as to their effects on workers' performance (B) investigating the effects that changes in working conditions would have on workers' performance (C) for investigating what are the effects in workers' performance that changes in working conditions would cause (D) that investigated changes in working conditions' effects on workers' performance (E) to investigate what the effects changes in working conditions would have on workers' performance

(b) is the best choice here. (a) is vague because it's overly indirect: the meaning of "investigate changes ... as to their effects" is unclear. what's more, it's probably considered unidiomatic as well, at least in this sort of context. (b) = correct the participle "investigating" follows "experiments" immediately. no filler words are necessary; this is good concision. the wording is clear; there are no awkward double possessives, etc., as in some of the other choices. "would" is used properly here, as a past-tense form of "will". (i.e., if this sentence were translated into the present tense, it would read "...that changes ... will have") (c) is ridiculously wordy; there's no way you should give this choice any serious consideration. if you don't realize pretty quickly that this choice is wrong, you should go back and read through a bunch of correct OG answers, trying to internalize the sights and sounds (the "vibe") of the correct answers. (d) "changes in working conditions' effects" is at best awkward and vague, and at worst ambiguous: the intended meaning is the effects of the changes, but this sentence seems to indicated the effects of the conditions themselves. in other words, a literal reading of this sentence seems to indicate that the conditions themselves haven't changed - only their effects have. that's not the intended meaning of the original. (e) "what the effects" is ungrammatical. also, in constructions of this sort, "what" is generally redundant / unnecessary; it's better merely to say "to investigate X" rather than to say "to investigate what X is" (or other such wordy construction).

When drive-ins were at the height of their popularity in the late 1950's, some 4,000 existed in the United States, but today there are less than one-quarter that many. A. there are less than one-quarter that many B. there are fewer than one-quarter as many C. there are fewer than one-quarter of that amount D. the number is less than one-quarter the amount E. it is less than one-quarter of that amount

*Countable: many, several, one, two, each, every, a number of, few Non-Countable: less, amount, much, hardly any, great,* Less and Amount are incorrect usage for countable nouns. Hence, A, C,D,E are wrong. Answer: B

if then

1. If you practice at something, [then] you will improve at it. Furthermore, notice that when a "then" is omitted, "if" does not have to come at the beginning of the sentence: 2. You will improve at something if you practice it. Now that you have the basic form, let's look at the different types of "if...then" constructions. The "General Rule" If...Then The first type of "if...then" construction implies a generality. The verb tense takes the form "if present tense, then present tense." Example: If you swim after you eat, you get a stomach ache. This type of "if...then" construction is identical to the use of the phrase of "whenever" to imply a generality: Whenever you swim after you eat, you get a stomach ache. The "General Rule" If...Then with Uncertainty These "if...then" constructions imply uncertainty with words like "can" or "may." They follow the form "If present tense, then can/may." Example: If John studies enough, he may pass the class. The "Particular Instance" in the Future These "if...then" constructions describe particular cases that will happen in the future. They follow the form "if present tense, then future tense." Example: If Susie doesn't show up to class tomorrow, she will fail the class. The "Particular Instance" in the Present These are very similar to the ones described above, but instead of using the future tense, these constructions take the form "if present perfect, then future tense." Example: If Susie has eaten shellfish, she will have a severe allergic reaction. The "Unlikely" Case (Future) If you want to express that something is unlikely to happen in the future, you must use the hypothetical subjunctive mood and the conditional. The hypothetical subjunctive, which often uses "if" or "as though," indicates conditions that are not likely to happen (e.g. The skilled swimmer glided through the sea as though he were a dolphin). The conditional tense uses "would" to show the result of an unlikely condition. This 'if...then' construction takes the form "if hypothetical subjunctive, then conditional." Example: If I were you, I would look for a job. Notice that the first clause indicates an unlikely idea (certainly "I" cannot be "you"). Instances that Did Not Take Place (past) To express hypothetical outcomes of instances that never took place, use the form "if past perfect, then conditional perfect." Example: If Peter had finished his homework, he would have passed the class. You can also use the

Modifier

1. When verb-ing modifier is separated from the clause using a comma, then this modifier modifies the preceding clause. 2. When verb-ing modifier is not separated from the clause using a comma, then it modifies the preceding noun. 3. When verb-ing modifier is placed in the beginning of the sentence followed by a comma, then it may modify either the subject of the clause or the entire clause, depending upon the context of the sentence. 4. Verb-ed modifier modifies the preceding noun or the noun phrase. 5. When verb-ed modifier is placed in the beginning of the clause followed by a comma, then it modifies the subject of the clause. V-ing结构在句子中可以做定语,状语。 1) 放在句子末尾时(逗号隔开),如果前面是主谓宾结构,如果同时时态上与主句搭配合理,V-ing结构作状语,作为: ① 伴随动作,表与主句动词同时发生的动作 ② 伴随结果,表主句动作带来的结果。 此时注意其逻辑主语的判断:其所修饰分句的动作发出者。 2) 如果前面是主系表结构(逗号隔开),由于系动词是一个状态不存在"被伴随",但是V-ing结构依然修饰的是整个主系表结构,所以后面的V-ing结构也可以被解读为修饰前面的名词成分—主语(如果宾语是名词,也一同修饰,因为主系表结构中主语和宾语所指是同一事物--见OG10-39, OG11-24中对正确选项C的解释)。

Modifier verb-ed and verb-ing modifier

1. When verb-ing modifier is separated from the clause using a comma, then this modifier modifies the preceding clause. 2. When verb-ing modifier is not separated from the clause using a comma, then it modifies the preceding noun. 3. When verb-ing modifier is placed in the beginning of the sentence followed by a comma, then it may modify either the subject of the clause or the entire clause, depending upon the context of the sentence. 4. Verb-ed modifier modifies the preceding noun or the noun phrase. 5. When verb-ed modifier is placed in the beginning of the clause followed by a comma, then it modifies the subject of the clause. http://gmatclub.com/forum/verb-ed-modifiers-vs-verb-ing-modifiers-125611.html

Unlikely hypothetical subjunctive-unlikely in the future-动词怎么用 case that never happened, 动词怎么用

1.If sophie ate pizza tomorrow, then she would become ill. 2. if sophie had eaten pizza yesterday, then she would have become ill.

整除 余数

2 4 8 2-看最后一位余数 4-最后两位余数 8-最后三位余数 3,9 把每一位都加起来,除以3或者9出来余几就是几 6-考虑被2 3整除的情况 5-最后一位余数 11-错位相加在相加之后除,看能不能整除 14916---1+9+6-(4+1)=11

108. One of the earliest known birds with a beak and contour feathers, Confuciusornis sanctus, with large clawlike "thumbs" on its wings, which probably helped them to climb up to a launching position for flight. A. with large clawlike "thumbs" on its wings, which probably helped them to B. with large clawlike "thumbs" on their wings, which probably helped it to C. had large clawlike "thumbs" on its wings, which probably helped them D. had large clawlike "thumbs" on its wings, probably to help it

A and B are wrong because "with a beak and contour feathers, Confuciusornis sanctus, with large clawlike..." is awkward. If it was going to keep this pattern, then it shouldn't separate beak, feathers and clawlike "thumbs".... they should all be listed together. So this rules out A and B because "had bla blah blah..." is correct. E is out because "their" (plural) doesn't agree with birds/Confuciusornis sanctus (singular; "birds" here is singular). So E is gone. Now, only left with C and D. Your idea of "probably helps it..." or "probably helping..." would work, IF either of those were an option. But their not. So we're left at choosing between C "...which probably helped them" or D "probably to help it". In C, "them" (plural) doesn't agree with "Confuciusornis sanctus" (singular), so C is eliminated. Only thing left is D, which works just fine ("it" is singular and refers back to the bird which is also singular). Although D may not be the way I would say this, after POE, it's clear that D is the only clear and concise answer choice.

With surface temperatures estimated at minus 230 degrees Farenheit, Jupiter's moon Europa has long been considered far too cold to support life, and, with 60 square miles of water thought to be frozen from top to bottom A) Europa has long been considered far too cold to support life, and with B) Europa has long been considered far too cold to support life, its C) Europa has long been considered as far too cold to support life and has D) Europa, long considered as far too cold to support life, and its E) Europa, long considered to be far too cold to support life, and to have

A) Europa has long been considered far too cold to support life, and with And is not required here. If we use AND then there should be a verb to support parallelism as in - Europa has long been considered far too cold to support life, AND, with 60 square miles of water IS thought to be frozen from top to bottom B) Europa has long been considered far too cold to support life, its Correct answer. Think of it like this: Europa, its 60 square miles of water thought to be frozen from top to bottom, has long been considered far too cold to support life. This is a type of modifier I think although I'm not sure what the construction is called. C) Europa has long been considered as far too cold to support life and has 'Considered as' is wrong. D) Europa, long considered as far too cold to support life, and its 'Considered as' is wrong. This sentence is a fragment, it does not have a verb - considered is not a proper verb, its a participle. E) Europa, long considered to be far too cold to support life, and to have This sentence is a fragment, it does not have a verb - considered is not a proper verb, its a participle.

As the former chair of the planning board for 18 consecutive years and a board member for 28 years, Joan Philkill attended more than 400 meetings and reviewed more than 700 rezoning applications. A. As the former B. The former C. Former D. She was E. As the

A. As the former PROBLEM: The "as" part is correct (See answer choice E), but you can't be a former chair for 18 years. Also, she didn't do these things AS the FORMER chair. She did them AS the chair, and NOW she is the former chair. B. The former PROBLEM: Technically, this could be called okay, because we have a really long noun modifier ("the former chair...") modifying another noun, "Joan Philkill." However, answer choice A simply makes more sense, considering the definition of "As" given above. C. Former PROBLEM: We need the article "the" here. We use it to refer to a person or thing that is unique. Obviously we're talking about a unique chair of the board here (Joan). D. She was PROBLEM: Now, we end up with two independent clauses separated by a comma. This isn't allowed. E. As the ANSWER: The preposition "as" can be used to to mean "during the time of being (the thing specified)." For example, I could say "As a child, I was often sick." In this sentence, we're told that "During the time of being the former chair...Joan attended more than 400 meetings...

Exceptions to the Modifier Touch Rule

As with almost anything in grammar, the Modifier Touch Rule is not a mathematically precise rule, and admits of exceptions. The BIG exception involves the distinction of vital vs. non-vital noun modifiers. A vital noun modifier always has logical priority over a non-vital noun modifier, and therefore could prevent a non-vital modifier from "touching" the noun by coming between the non-vital modifier and the noun. 4) In the last decades of his life, Rimsky-Korsakov produced a massive book on orchestration, which is still read by composition students today. The modifier "which is still ..." doesn't modify the noun "orchestration" (which is something that can't be "read") — instead, it modifies "book", and it's fine that it doesn't touch "book", because the modifier "on orchestration" is a vital noun modifier — that is, it narrows down and identifies the indefinite noun "book." Other examples of exceptions to the Touch Rule involve a short set of words, such as an example phrase or a short intransitive(不及物) verb phrase, that are correctly placed between a noun and its modifier. 5) The most expensive component of any catalytic convertor is the small quantity of precious metal, such as platinum or rhodium, which acts as the catalyst. 6) Last week, the senator resigned who made the disparaging remark about older women. In #5, the modifier "which acts as the catalyst" modifies "precious metal," even though the short example phrase comes between. In #6, the modifier "who made ..." modifies "the senator," even though a short verb intervenes.

86. (Book Question: 756)To map Earth's interior, geologists use a network of seismometers to chart seismic waves that originate in the earth's crust and ricochet around its interior, most rapidly traveling through cold, dense regions and slower through hotter rocks. A. interior, most rapidly traveling through cold, dense regions and slower B. interior, which travel most rapidly through cold, dense regions, and more slowly C. interior, traveling most rapidly through cold, dense regions and more slowly D. interior and most rapidly travel through cold, dense regions, and slower E. interior and that travel most rapidly through cold, dense regions and slower

C is not modifying the interior but it is modifying the whole preceding phrase. 记得是前一个,不是Geologist slower不对,不合rapidly链接 the comma + ing phrase can be used to show cause and effect relationship as well as to give extra info about the preceding phrase. Here the modifier is performing the latter part. And between slower and more slowly we require adverb to modify travelling. So more slowly is correct Correct Answer: C Selected Answer: E Grammatical construction; Parallelism This sentence explains in detail an activity of geologists (using seismometers to chart waves), focusing primarily on the object, seismic waves. A description of these waves is developed in a relative clause (that originate . . . hotter rocks) that contains a compound verb phrase (originate . . . ricochet . . .). The action, ricochet, is further described in a participial phrase in which traveling . . . is then further described in a comparison of travel speeds in cold and hot regions of Earth's crust. A The two expressions of comparison should be parallel. Because most rapidly is placed before the verb, it appears to modify the entire ensuing phrase, including slower. This and the contrast between the forms of rapidly and slower make the comparisons nonparallel. Some usage advisers consider slower to be only an adjective. Although slower is sometimes used as an adverb, that usage would be more appropriate with the parallel faster. The stark contrast between this typically adjectival form and the clearly adverbial ly form is somewhat jarring. B The referent of the relative pronoun which is unclear. C Correct. The modifiers are parallel and correctly positioned in relation to the verb. D This version of the sentence offers travel as a compound verb parallel with originate and ricochet rather than as a description of how the waves ricochet. It has the same problems with parallelism as (A). E Adding a relative clause and that . . . makes this sentence wordy and awkward.

21. (Book Question: 675)Diabetes, together with its serious complications, ranks as the nation's third leading cause of death, surpassed only by heart disease and cancer. A. ranks as the nation's third leading cause of death, surpassed only B. rank as the nation's third leading cause of death, only surpassed C. has the rank of the nation's third leading cause of death, only surpassed D. are the nation's third leading causes of death, surpassed only E. have been ranked as the nation's third leading causes of death, only surpassed

Correct Answer: A Selected Answer: A Agreement; Logical predication This sentence correctly matches the singular verb, ranks, with the singular subject, diabetes, and uses the present tense to indicate a current situation. The phrase following diabetes is set off by a pair of commas, indicating that it is descriptive information that may be dropped from the sentence; it is not a part of the subject. Only is placed with precision next to the group of words it actually limits, by heart disease and cancer. Placed before surpassed, only would more ambiguously limit surpassed. A Correct. In the original sentence, the subject and verb agree, and the proper tense is used; only is correctly placed next to the phrase it limits. B Rank does not agree with diabetes; only limits surpassed rather than by heart disease and cancer. C Has the rank of is wordy and unidiomatic; only limits surpassed rather than by heart disease and cancer. D Construction are . . . causes does not agree with diabetes. E Construction have been ranked . . . causes does not agree with diabetes and uses the wrong verb tense; only limits surpassed rather than by heart disease and cancer. Diabetes ranks as the nation's third leading cause of the death, surpassing only by heart disease and cancer. The first sentence is grammatically as well as logically correct. But the second is not grammatically correct. The first sentence can be rewritten as: Diabetes is the nation's third leading cause of the death that is surpassed only by heart disease and cancer. Notice that the "that" clause is written in passive voice because diabetes is not the doer of the action "surpass". It is "heart disease and cancer" that are the doer of this action. In the second sentence, "surpassing" modifies the preceding clause and hence associates with the subject diabetes. So if we say that Diabetes is X, surpassing only by Y and Z, it will be wrong because it is not the correct grammatical structure. Use of "by" is ungrammatical in this construction. If we remove "by" from here, then the intended meaning of the sentence will change. The sentence will then mean that Diabetes surpasses "heart diseases and cancer" but it is actually the other way round and that is why diabetes is "the nation's third leading cause of death".

34. (Book Question: 266)Certain pesticides can become ineffective if used repeatedly in the same place; one reason is suggested by the finding that there are much larger populations of pesticide-degrading microbes in soils with a relatively long history of pesticide use than in soils that are free of such chemicals. A. Certain pesticides can become ineffective if used repeatedly in the same place; one reason is suggested by the finding that there are much larger populations of pesticide-degrading microbes in soils with a relatively long history of pesticide use than in soils that are free of such chemicals. B. If used repeatedly in the same place, one reason that certain pesticides can become ineffective is suggested by the finding that there are much larger populations of pesticide-degrading microbes in soils with a relatively long history of pesticide use than in soils that are free of such chemicals. C. If used repeatedly in the same place, one reason certain pesticides can become ineffective is suggested by the finding that much larger populations of pesticide-degrading microbes are found in soils with a relatively long history of pesticide use than those that are free of such chemicals. D. The finding that there are much larger populations of pesticide-degrading microbes in soils with a relatively long history of pesticide use than in soils that are free of such chemicals is suggestive of one reason, if used repeatedly in the same place, certain pesticides can become ineffective. E. The finding of much larger populations of pesticide-degrading microbes in soils with a relatively long history of pesticide use than in those that are free of such chemicals suggests one reason certain pesticides can become ineffective if used repeatedly in the same place.

Correct Answer: A Selected Answer: B Logical predication; Rhetorical construction The sentence is correctly constructed; it has two independent clauses connected by a semicolon. If used repeatedly in the same place clearly and correctly modifies certain pesticides. A Correct. The sentence is correctly constructed; the modifier if used repeatedly in the same place is correctly placed. B If used repeatedly in the same place modifies one reason when it should modify certain pesticides. C If used repeatedly in the same place modifies one reason when it should modify certain pesticides. The absence of in in the phrase than those . . . makes the comparison unclear. D If used repeatedly in the same place ambiguously modifies one reason when it should clearly modify certain pesticides. E The comparison the finding of much larger populations . . . than in those that . . . is improperly constructed in a way that makes the finding appear to refer awkwardly to a discovery of larger populations rather than to a research conclusion about the presence of such populations.

19. (Book Question: 742)The nineteenth-century chemist Humphry Davy presented the results of his early experiments in his "Essay on Heat and Light," a critique of all chemistry since Robert Boyle as well as a vision of a new chemistry that Davy hoped to found. A. a critique of all chemistry since Robert Boyle as well as a vision of a B. a critique of all chemistry following Robert Boyle and also his envisioning of a C. a critique of all chemistry after Robert Boyle and envisioning as well D. critiquing all chemistry from Robert Boyle forward and also a vision of E. critiquing all the chemistry done since Robert Boyle as well as his own envisioning of

Correct Answer: A Selected Answer: B Parallelism; Rhetorical construction The main objective of the sentence is to describe "Essay on Heat and Light" as Davy's presentation of his own experiments and to further explain that the essay served as both a critique of previous chemistry and a vision of a new kind of chemistry. The clearest, most effective form for providing this explanation of the essay's function is to make critique and vision both appositives of "Essay on Heat and Light," and to present them in a parallel structure. A Correct. The phrases describing the essay's function are presented in parallel form. B Critique and his envisioning are not parallel; the phrase and also his envisioning is unnecessarily wordy; it is also unclear to whom his refers. C The two descriptors are not parallel. D The two descriptors are not parallel. E The meaning is confused in the assertion that Davy critiqued his own vision of chemistry.

48. (Book Question: 258)Even though the overall consumer price index did not change in April, indicating the absence of any general inflation or deflation, prices in several categories of merchandise have fallen over the last several months. A. April, indicating the absence of any general inflation or deflation, prices in several categories of merchandise have fallen B. April, indicating that any general inflation or deflation were absent, prices in several categories of merchandise fell C. April and indicated that absence of any general inflation or deflation, prices in several categories of merchandise fell D. April, having indicated the absence of any general inflation or deflation, prices in several categories of merchandise fell E. April, which indicated that any general inflation or deflation were absent, prices in several categories of merchandise have fallen

Correct Answer: A Selected Answer: B Rhetorical construction; Agreement Coordinated noun phrases in which singular nouns are linked by or are considered singular, so when the phrase any general inflation or deflation is a subject, it requires a singular verb. One of the answer choices incorrectly uses the word that. Another phrasing problem is with indicating/indicated. Indicating works well as a verb form in the options where it occurs. A Correct. Any general inflation or deflation is not a subject (it functions as the object of the preposition of), so there is no potential agreement problem. B Were is incorrect as the agreeing form of be; it should be is. C If that is taken as a demonstrative adjective modifying absence, it is inappropriate (the word the is required); if it is meant as a subordinating conjunction, it is incorrect because it is not followed by a clause. D Having indicated is unclear and unnecessarily long. It appears to say, somewhat illogically, that the indication occurred at some unspecified time prior to the lack of change in April. Indicating works well alone and would be a preferable verb form here. E Were here could only be meant either as a plural past-tense verb or as a singular subjunctive-mood verb (appropriate only in certain conditional contexts); in either case it is incorrect. Also, indicating works well, and the which-clause is unnecessary.

102. (Book Question: 763)A mutual fund having billions of dollars in assets will typically invest that money in hundreds of companies, rarely holding more than one percent of the shares of any particular corporation. A. companies, rarely holding more than one percent B. companies, and it is rare to hold at least one percent or more C. companies and rarely do they hold more than one percent D. companies, so that they rarely hold more than one percent E. companies; rarely do they hold one percent or more

Correct Answer: A Selected Answer: C Agreement; Logical predication The participial phrase starting with rarely holding is predicated of the main subject a mutual fund. It elaborates on the effect of the main clause verb: since a mutual fund invests in hundreds of companies, it rarely holds more than one percent in any particular corporation. A Correct. The participle holding in the embedded clause correctly refers to a mutual fund. It also correctly expresses the cause-and-effect relationship between investing in many companies and holding little in each company. B The antecedent of it is rare to hold is not clear. The use of it is rare instead of rarely could be misleading. C The use of and between the clauses makes them both main clauses. Thus, the cause-and-effect relationship between investing and holding is lost. The referent of they is unclear. It makes no sense to suppose that it refers to the hundreds of companies. Since it presumably refers to a mutual fund, it should be singular. D The pronoun they refers to a mutual fund and thus should be singular. E The pronoun they refers to a mutual fund and thus should be singular.

13. (Book Question: 284)Schistosomiasis, a disease caused by a parasitic worm, is prevalent in hot, humid climates, and it has become more widespread as irrigation projects have enlarged the habitat of the freshwater snails that are the parasite's hosts for part of its life cycle. A. the freshwater snails that are the parasite's hosts for part of its life cycle B. the freshwater snails that are the parasite's hosts in part of their life cycle C. freshwater snails which become the parasite's hosts for part of its life cycles D. freshwater snails which become the hosts of the parasite during the parasite's life cycles E. parasite's hosts, freshwater snails which become their hosts during their life cycles

Correct Answer: A Selected Answer: C Rhetorical construction; Agreement This sentence explains the increased incidence of schistosomiasis as a consequence of the enlarged habitat of the kind of freshwater snails that host the parasitic worm responsible for the disease. The definite article is necessary before freshwater snails because the sentence identifies a particular type of snail, namely, those that host the parasite. The correct preposition to express duration in combination with host is for, not in. As the parasite is referred to as singular, the possessive pronoun in the final phrase must also be singular. A Correct. The sentence is clear with all pronouns and verbs in agreement. B The preposition in is inappropriate for expressing duration; the plural possessive pronoun their does not agree with the singular antecedent parasite. C A definite article should precede freshwater snails to identify a particular type of snail; the plural cycles is inappropriate because its refers to a singular parasite, which only has one life cycle. D A definite article is needed before freshwater snails; repetition of the word parasite makes the final phrase unnecessarily wordy; cycles should be singular. E The repetition of hosts makes the final phrase unnecessarily wordy; cycles should be singular; their hosts should be its hosts; the referent for the second appearance of their is unclear—does it refer to snails or the parasite?

49. (Book Question: 285)Floating in the waters of the equatorial Pacific, an array of buoys collects and transmits data on long-term interactions between the ocean and the atmosphere, interactions that affect global climate. A. atmosphere, interactions that affect B. atmosphere, with interactions affecting C. atmosphere that affects D. atmosphere that is affecting E. atmosphere as affects

Correct Answer: A Selected Answer: D Grammatical construction; Agreement The underlined portion of the sentence is an appositive, a terminal noun phrase restating the kind of data being collected and providing additional information about it. This is a clear and economical way to provide the extra information. A Correct. The sentence is grammatically correct and logically coherent. B The prepositional phrase with . . . has no clear noun or noun phrase to attach to and is therefore ungrammatical. C Using the restrictive that after atmosphere illogically suggests that there are many atmospheres to differentiate from and the one in question in this sentence is the one affecting global climate. D The restrictive that also follows atmosphere as in answer C. E The phrase as affects global climate functions as an adverb, but there is no verb for it to modify.

24. (Book Question: 208)The gyrfalcon, an Arctic bird of prey, has survived a close brush with extinction; its numbers are now five times greater than when the use of DDT was sharply restricted in the early 1970's. A. extinction; its numbers are now five times greater than B. extinction; its numbers are now five times more than C. extinction, their numbers now fivefold what they were D. extinction, now with fivefold the numbers they had E. extinction, now with numbers five times greater than

Correct Answer: A Selected Answer: E Agreement; Diction; Logical predication The original sentence contains no errors. The semicolon correctly connects the closely related ideas in the two independent clauses. The gyrfalcon is the antecedent for its in the second phrase. A Correct. The original sentence correctly uses a singular pronoun, its, to refer to the singular antecedent gyrfalcon, and it properly uses the construction its numbers are . . . greater than. B The use of more instead of greater inappropriately implies that there are now more numbers, rather than more gyrfalcons. C The pronoun their is plural, and thus incorrect, since the antecedent gyrfalcon is singular. Fivefold what they were is awkward and nonstandard and implies that there are now more numbers, rather than more gyrfalcons. D The pronoun they is plural, and thus incorrect, since the antecedent gyrfalcon is singular. The comma introduces a confusing phrase seeming to modify extinction. Fivefold the numbers they had is awkward and nonstandard and implies that there are now more numbers, rather than more gyrfalcons. E The comma introduces a confusing phrase seeming to modify extinction.

26. (Book Question: 772)Heating-oil prices are expected to be higher this year than last because refiners are paying about $5 a barrel more for crude oil than they were last year. A. Heating-oil prices are expected to be higher this year than last because refiners are paying about $5 a barrel more for crude oil than they were B. Heating-oil prices are expected to rise higher this year over last because refiners pay about $5 a barrel for crude oil more than they did C. Expectations are for heating-oil prices to be higher this year than last year's because refiners are paying about $5 a barrel for crude oil more than they did D. It is the expectation that heating-oil prices will be higher for this year over last because refiners are paying about $5 a barrel more for crude oil now than what they were E. It is expected that heating-oil prices will rise higher this year than last year's because refiners pay about $5 a barrel for crude oil more than they did

Correct Answer: A Selected Answer: E Rhetorical construction; Idiom The sentence connects a comparison between this year's and last year's heating-oil prices with a comparison between this year's and last year's crude-oil prices. The most efficient, parallel expression of those comparisons is to use two comparative expressions, higher than and more than. A Correct. This sentence expresses the comparison in succinct, parallel phrases. B The comparative form, higher, anticipates the comparative term than, not over; in the second clause, the comparative terms more than should immediately follow $5 a barrel. C Expectations are for . . . is an unnecessarily wordy and indirect expression; the possessive year's is not parallel with the adverbial phrase this year. D It is the expectation that . . . is wordy and awkward; for and what are unnecessary. E It is expected that . . . is wordy and awkward; the possessive last year's does not parallel the adverbial phrase this year.

Flag for Review 28. (Book Question: 294)The commission proposed that funding for the park's development, which could be open to the public early next year, is obtained through a local bond issue. A. that funding for the park's development, which could be open to the public early next year, is B. that funding for development of the park, which could be open to the public early next year, be C. funding for the development of the park, perhaps open to the public early next year, to be D. funds for the park's development, perhaps open to the public early next year, be E. development funding for the park, which could be open to the public early next year, is to be

Correct Answer: B Selected Answer: A Logical predication; Verb form Which modifies the noun that precedes it; in this sentence, the clause beginning with which illogically refers to development rather than the park. This error can be corrected by substituting development of the park (which follows park) for park's development (which follows development). When a verb such as recommend, request, or propose is used in the main clause, the verb following that in the subordinate clause is subjunctive (be) rather than indicative (is). A Which modifies development instead of park. Be is required, not is. B Correct. In this sentence, which clearly modifies park; the subjunctive be correctly follows proposed that. C Be is required, not the infinitive to be. D That is omitted, making the construction awkward and unclear. The phrase modifies development, not park. E Development funding distorts the meaning. Be is required, not is to be.

112. (Book Question: 227)Some buildings that were destroyed and heavily damaged in the earthquake last year were constructed in violation of the city's building code. A. Some buildings that were destroyed and heavily damaged in the earthquake last year were B. Some buildings that were destroyed or heavily damaged in the earthquake last year had been C. Some buildings that the earthquake destroyed and heavily damaged last year have been D. Last year the earthquake destroyed or heavily damaged some buildings that have been E. Last year some of the buildings that were destroyed or heavily damaged in the earthquake had been

Correct Answer: B Selected Answer: B Diction; Verb form The buildings cannot be both destroyed and heavily damaged at the same time; they must be one or the other. The ideas of this sentence are most clearly expressed using two verb tenses: the simple past, were, for the earthquake occurring last year; and the past perfect, had been, for the time prior to that when the buildings were constructed. A The buildings are illogically said to be both destroyed and damaged. B Correct. This sentence properly states that the buildings were either destroyed or damaged and clarifies that they had been constructed before the earthquake struck. C Buildings cannot be both destroyed and damaged. The verb tense makes it seem that they were constructed after the earthquake. D The verb tense illogically indicates that the buildings have been constructed since the earthquake. E This structure indicates that construction of the buildings, rather than the earthquake, occurred last year.

Some anthropologists believe that the genetic homogeneity evident in the world's people is the result of a "population bottleneck"—at some time in the past our ancestors suffered an event, greatly reducing their numbers and thus our genetic variation. A. at some time in the past our ancestors suffered an event, greatly reducing their numbers B. that at some time in the past our ancestors suffered an event that greatly reduced their numbers C. that some time in the past our ancestors suffered an event so that their numbers were greatly reduced, D. some time in the past our ancestors suffered an event from which their numbers were greatly reduced E. some time in the past, that our ancestors suffered an event so as to reduce their numbers greatly,

Correct Answer: B Selected Answer: B Grammatical construction; Parallelism The underlined part of this sentence is an explanatory rewording of the clause that follows believe. Scientists believe that X—[in other words,] that Y. In this construction, X and Y are parallel clauses. A The omission of that after the dash makes the function of the final clause unclear. The structure makes that clause appear to be an awkward and rhetorically puzzling separate assertion that the writer has appended to the prior claim about what the anthropologists believe. The agent or cause of reducing is unclear. B Correct. Repetition of that effectively signals the paraphrasing of the belief. C The preposition at before some time is missing; without at the adverb sometime would be needed instead of this two-word noun phrase. The modifier of event is expressed with a wordy passive construction, which destroys the parallelism between it and what follows. D Repetition of that signals the paraphrasing of the belief and is therefore needed. The preposition at before some time is missing. The modifier of event is expressed with a wordy passive construction, which destroys the parallelism between it and what follows. E That is repeated in the paraphrase, but in the wrong place. A possible, and absurd, reading of this version is that our ancestors suffered an event in order to willfully reduce their own numbers and thus our genetic variation. -后面跟的是一个modifier,dash解释unambiguous meaning The dash (-) is a flexible punctuation mark that the GMAT occasionally employs. You canuse a dash as an emphatic comma, semicolon, or colon. For example: Right: By January 2, 2000, the so-called "Y2K problem" was already widely considered a joke - although the reason for the non-event was the prior corporate and governmental investment in countermeasures.In the case above, either a comma or a dash would be correct. Sometimes, a dash helps to maintain an unambiguous meaning. For instance, compare these two sentences: Wrong: My three best friends, Danny, Enrico, and Joey, and I went skiing. Right: My three best friends- Danny, Enrico, and Joey- and I went skiing. If you used commas in this sentence, the reader might think that seven people were going skiing (you, your three best friends, and Danny, Enrico, and Joey) rather than four. A at some time in the past our ancestors suffered an event, greatly reducing their numbers ------ A hyphen is not the tool to connect two ICs. B that at some time in the past our ancestors suffered an event that greatly reduced their numbers -------- 'that ' is the right connector ---correct choice. C that sometime in the past our ancestors suffered an event so that their numbers were greatly reduced ------- altered notion; it looks as if the ancestors suffered a calamity in order to reduce their numbers. D some time in the past our ancestors suffered an event from which their numbers were greatly reduced -------- conjugation of two ICs with hyphenation is wrong. E some time in the past, that our ancestors suffered an event so as to reduce their numbers greatly, ------ same as in D

33. (Book Question: 791)Found only in the Western Hemisphere and surviving through extremes of climate, hummingbirds' range extends from Alaska to Tierra del Fuego, from sea-level rain forests to the edges of Andean snowfields and ice fields at altitudes of 15,000 feet. A. Found only in the Western Hemisphere and surviving through extremes of climate, hummingbirds' range extends B. Found only in the Western Hemisphere, hummingbirds survive through extremes of climate, their range extending C. Hummingbirds, found only in the Western Hemisphere and surviving through extremes of climate, with their range extending D. Hummingbirds, found only in the Western Hemisphere and surviving through extremes of climate, their range extends E. Hummingbirds are found only in the Western Hemisphere, survive through extremes of climate, and their range extends

Correct Answer: B Selected Answer: B Logical predication; Grammatical construction This sentence makes three points about hummingbirds: they live in the Western Hemisphere, they survive extreme climates, and their range is wide and varied. Hummingbirds, not hummingbirds' range, should be the subject of the sentence. A The opening modifier, found . . . and surviving . . . should modify hummingbirds, not, as it does, hummingbirds' range. B Correct. Hummingbirds is the subject of the sentence, and the use of the absolute phrase their range extending appropriately connects the final clause to the rest of the sentence. C This version of the sentence has no main verb for the subject Hummingbirds and as a result is ungrammatical. D Like (C), this version of the sentence has no main verb for the subject Hummingbirds. E This version of the sentence begins with the subject Hummingbirds attached to a pair of verb phrases (are found . . . and survive . . .) followed by the conjunction and, which suggests that what follows should also be a verb phrase; instead, a new subject is introduced, their range. Replacing the comma before survive with and would render the sentence acceptable. Found only in the Western Hemisphere, hummingbirds survive through extremes of climate, their range extending.. you are correct that Found only in the Western Hemisphere modifies hummingbirds.. but their range extending.. modifies the entier clause prior to it-hummingbirds survive through extremes of climate.. Due to this capability to survive in extreme climates, their range extends in varities of location..

29. (Book Question: 758)Prices at the producer level are only 1.3 percent higher now than a year ago and are going down, even though floods in the Midwest and drought in the South are hurting crops and therefore raised corn and soybean prices. A. than a year ago and are going down, even though floods in the Midwest and drought in the South are hurting crops and therefore raised B. than those of a year ago and are going down, even though floods in the Midwest and drought in the South are hurting crops and therefore raising C. than a year ago and are going down, despite floods in the Midwest and drought in the South, and are hurting crops and therefore raising D. as those of a year ago and are going down, even though floods in the Midwest and drought in the South hurt crops and therefore raise E. as they were a year ago and are going down, despite floods in the Midwest and drought in the South, and are hurting crops and therefore raising

Correct Answer: B Selected Answer: B Logical predication; Verb form The sentence as written makes an illogical comparison between prices at the producer level and a time period (a year ago); surely the intended comparison is between such prices now and those of a year ago. The clause at the end of the sentence states that flooding and a drought are hurting crops, and as a result of this, they have raised prices of certain crops. Are hurting is in the present progressive tense, indicating an ongoing process; raised is in the simple past tense, indicating a completed action. It would be more appropriate to use the present progressive tense here as well, [are] raising. A The first part of this version of the sentence illogically compares prices to a time; the second part of the sentence indicates that a completed action (raised . . . prices) results from an ongoing present condition (floods . . . and drought . . . are hurting crops). B Correct. This version of the sentence makes an appropriate comparison (between prices now and those of a year ago), and uses tenses in an appropriate way. C Like (A), this version of the sentence illogically compares prices to time past. Furthermore, in this version, Prices is the subject not only for the verb are but also for the verbs are hurting and [are] raising, which makes no sense. D The comparative adjective higher requires the comparative term than instead of as; the tenses of the verbs in the latter half of the sentence, hurt (simple past) and raise (simple present), do not work together logically. E Like (D), this version inappropriately uses as instead of than with higher. Furthermore, like (C), in this version [p]rices is the subject not only for the verb are but also for the verbs are hurting and [are] raising.

101. (Book Question: 244)Recently discovered fossil remains strongly suggest that the Australian egg-laying mammals of today are a branch of the main stem of mammalian evolution rather than developing independently from a common ancestor of mammals more than 220 million years ago. A. rather than developing independently from B. rather than a type that developed independently from C. rather than a type whose development was independent of D. instead of developing independently from E. instead of a development that was independent of

Correct Answer: B Selected Answer: C Idiom; Parallelism The original point is that the mammals mentioned are thought to be an offshoot of the main stem of mammalian evolution and not a descendent of a common ancestor of [all] mammals. This sentence makes a contrast using the construction x rather than y or x instead of y; x and y must be parallel in either case. The mammals are (x) a branch rather than (y); here y should consist of an article and a noun to match a branch. The second half of the contrast may be rewritten a type that developed independently from to complete the parallel construction. The idiom independently from is different in meaning from the idiom independent of; the logic of this sentence requires the use of independently from. A Developing independently from is not parallel to a branch. B Correct. This idiomatically correct sentence properly uses a type in parallel to a branch. C The verb developed is preferable to the awkward and wordy relative clause using the noun development; independent of distorts the original meaning. D Developing independently from is not parallel to a branch. E While a development may appear to parallel a branch, a development that was independent of . . . expresses a meaning contrary to that expressed in the original sentence. The verb developed is preferable to the noun development.

74. (Book Question: 736)Some anthropologists believe that the genetic homogeneity evident in the world's people is the result of a "population bottleneck"—at some time in the past our ancestors suffered an event, greatly reducing their numbers and thus our genetic variation. A. at some time in the past our ancestors suffered an event, greatly reducing their numbers B. that at some time in the past our ancestors suffered an event that greatly reduced their numbers C. that some time in the past our ancestors suffered an event so that their numbers were greatly reduced, D. some time in the past our ancestors suffered an event from which their numbers were greatly reduced E. some time in the past, that our ancestors suffered an event so as to reduce their numbers greatly,

Correct Answer: B Selected Answer: D Grammatical construction; Parallelism The underlined part of this sentence is an explanatory rewording of the clause that follows believe. Scientists believe that X—[in other words,] that Y. In this construction, X and Y are parallel clauses. A The omission of that after the dash makes the function of the final clause unclear. The structure makes that clause appear to be an awkward and rhetorically puzzling separate assertion that the writer has appended to the prior claim about what the anthropologists believe. The agent or cause of reducing is unclear. B Correct. Repetition of that effectively signals the paraphrasing of the belief. C The preposition at before some time is missing; without at the adverb sometime would be needed instead of this two-word noun phrase. The modifier of event is expressed with a wordy passive construction, which destroys the parallelism between it and what follows. D Repetition of that signals the paraphrasing of the belief and is therefore needed. The preposition at before some time is missing. The modifier of event is expressed with a wordy passive construction, which destroys the parallelism between it and what follows. E That is repeated in the paraphrase, but in the wrong place. A possible, and absurd, reading of this version is that our ancestors suffered an event in order to willfully reduce their own numbers and thus our genetic variation.

14. (Book Question: 781)Because there are provisions of the new maritime code that provide that even tiny islets can be the basis for claims to the fisheries and oil fields of large sea areas, they have already stimulated international disputes over uninhabited islands. A. Because there are provisions of the new maritime code that provide that even tiny islets can be the basis for claims to the fisheries and oil fields of large sea areas, they have already stimulated B. Because the new maritime code provides that even tiny islets can be the basis for claims to the fisheries and oil fields of large sea areas, it has already stimulated C. Even tiny islets can be the basis for claims to the fisheries and oil fields of large sea areas under provisions of the new maritime code, already stimulating D. Because even tiny islets can be the basis for claims to the fisheries and oil fields of large sea areas under provisions of the new maritime code, this has already stimulated E. Because even tiny islets can be the basis for claims to the fisheries and oil fields of large sea areas under provisions of the new maritime code, which is already stimulating

Correct Answer: B Selected Answer: D Logical predication; Grammatical construction In this sentence, the there are . . . that . . . construction contributes nothing more than unnecessary words. The sentence needs to make clear whether provisions or code is the subject of the main verb stimulated. A The there are . . . that . . . construction is unnecessarily wordy; in the predicate nominative instead of the subject position, provisions is not an obvious referent for the pronoun they. B Correct. In this sentence, the new maritime code is clearly the antecedent of it in the main clause and thus the subject of has already stimulated. C Under provisions of the new maritime code is a misplaced modifier, seeming to describe sea areas; the sentence does not make clear what is stimulating . . . disputes. D The referent of this is unclear. E The sentence is a fragment, opening with a dependent clause (Because . . . code) and concluding with a relative clause, but lacking a main, independent clause.

43. (Book Question: 203)A March 2000 Census Bureau survey showed that Mexico accounted for more than a quarter of all foreign-born residents of the United States, the largest share for any country to contribute since 1890, when about 30 percent of the country's foreign-born population was from Germany. A. the largest share for any country to contribute B. the largest share that any country has contributed C. which makes it the largest share for any country to contribute D. having the largest share to be contributed by any country E. having the largest share to have been contributed by any country

Correct Answer: B Selected Answer: D Logical predication; Rhetorical construction This sentence claims that the 2000 Census showed that at the time Mexico's contribution to the foreign-born population of United States residents exceeded that of any other country since 1890. It makes the comparison in an appositive that modifies more than a quarter of all foreign-born residents of the United States. A The phrase for any country to contribute makes the sentence wordy and indirect. B Correct. This form of the appositive is the most efficient way to express the comparison. Depending on when the sentence was written and what the writer intended to express, the verb form could be either had contributed or has contributed. The use of has contributed implies that, from the perspective of the sentence, the comparison between German-born U.S. residents and those from other countries still holds true. C The antecedents of the relative pronoun which and the pronoun it are ambiguous. Along with the prepositional phrase, the pronouns contribute wordiness and indirection. D This construction is awkward, wordy, and indirect, and the use of the present tense of the infinitive is inappropriate. E This construction is awkward, wordy, and indirect.

Thomas Eakins' powerful style and his choices of subject—the advances in modern surgery, the discipline of sport, the strains of individuals in tension with society or even with themselves—was as disturbing to his own time as it is compelling for ours. A. was as disturbing to his own time as it is B. were as disturbing to his own time as they are C. has been as disturbing in his own time as they are D. had been as disturbing in his own time as it was E. have been as disturbing in his own time as

Correct Answer: B Selected Answer: E Agreement; Verb form The compound subject of this sentence, style and choices, is followed by singular verbs, was and is, and a singular pronoun, it. The compound subject requires the plural verbs were and are and the plural pronoun they. A The verbs and pronoun are singular, but the subject is plural. B Correct. Verbs (were, are) and pronoun (they) agree with the plural subject in this sentence. C Has been is singular and illogically indicates that Eakins' time continues today. D Had been indicates a time anterior to some other past time; it was is singular and the wrong tense. E Have been illogically indicates that Eakins' time continues into the present day.

While it costs about the same to run nuclear plants as other types of power plants, it is the fixed costs that stem from building nuclear plants that makes it more expensive for them to generate electricity. A. While it costs about the same to run nuclear plants as other types of power plants, it is the fixed costs that stem from building nuclear plants that makes it more expensive for them to generate electricity. B. While the cost of running nuclear plants is about the same as for other types of power plants, the fixed costs that stem from building nuclear plants make the electricity they generate more expensive. C. Even though it costs about the same to run nuclear plants as for other types of power plants, it is the fixed costs that stem from building nuclear plants that makes the electricity they generate more expensive. D. It costs about the same to run nuclear plants as for other types of power plants, whereas the electricity they generate is more expensive, stemming from the fixed costs of building nuclear plants. E. The cost of running nuclear plants is about the same as other types of power plants, but the electricity they generate is made more expensive because of the fixed costs stemming from building nuclear plants.

Correct Answer: B Selected Answer: E Agreement; Logical predication The emphatic construction it is X that does Y (as in the phrase it is Jane who knows the answer) should be used only when there is a compelling reason to emphasize the doer of the action. In this sentence, the emphatic construction is used without good reason. A This sentence uses the emphatic structure it is . . . that without justification. The singular verb makes violates the agreement within the structure. The verb makes should agree with the notional subject (the fixed costs), not with the pronoun it. B Correct. This answer choice clearly and succinctly compares the two types of costs. C In addition to using the more cumbersome emphatic structure, this version violates the agreement within the structure. The verb should agree with the notional subject (the fixed costs), not with the pronoun it. D The preposition for is redundant in comparing the two objects of run. Since it is not clear what stemming . . . refers to, this is a dangling modifier. E The passive construction electricity . . . is made more expensive because of . . . is wordy and cumbersome. The preposition for is necessary in the comparison of the costs. A) The costs of running nuclear plants are compared with other types of plants. Incorrect. B) Correct. Compares the costs correctly and there is no pronoun ambiguity either. C) 'to run'....'as for' is ungrammatical and incorrect usage. Incorrect. D) Same issue as C. Incorrect. E) Same issue as A. Incorrect.

33. (Book Question: 274)Bluegrass musician Bill Monroe, whose repertory, views on musical collaboration, and vocal style were influential on generations of bluegrass artists, was also an inspiration to many musicians, that included Elvis Presley and Jerry Garcia, whose music differed significantly from his own. A. were influential on generations of bluegrass artists, was also an inspiration to many musicians, that included Elvis Presley and Jerry Garcia, whose music differed significantly from B. influenced generations of bluegrass artists, also inspired many musicians, including Elvis Presley and Jerry Garcia, whose music differed significantly from C. was influential to generations of bluegrass artists, was also inspirational to many musicians, that included Elvis Presley and Jerry Garcia, whose music was different significantly in comparison to D. was influential to generations of bluegrass artists, also inspired many musicians, who included Elvis Presley and Jerry Garcia, the music of whom differed significantly when compared to E. were an influence on generations of bluegrass artists, was also an inspiration to many musicians, including Elvis Presley and Jerry Garcia, whose music was significantly different from that of

Correct Answer: B Selected Answer: E Agreement; Rhetorical construction; Grammatical construction The original sentence logically intends to explain that Monroe's work influenced generations of artists in his own musical field and that he inspired many musicians in other musical fields. Who or what influenced or inspired whom must be more clearly stated. Additionally, the original sentence lacks precision, being overly wordy and using phrases that are not idiomatic. Concise and consistent verb forms, as well as the use of subordinate phrases rather than clauses, improve the precision of the sentence. A The phrase were influential on is wordy and is not idiomatic; the use of verb forms were (the predicate of repertory, views, and style) and was (the predicate of Monroe) is confusing. B Correct. The use of the concise verb forms of influenced and inspired simplifies and clarifies the sentence. The concise use of including avoids the pronoun error and unnecessary wordiness. C The subject and verb do not agree in repertory, views, and style . . . was (compound subject with singular verb). Was influential to and different . . . in comparison to are unnecessarily wordy. D There is incorrect subject-verb agreement in repertory, views, and style . . . was (compound subject with singular verb). Was influential to and in when compared to are unnecessarily wordy. The music of whom is cumbersome and stilted. E The phrase were an influence on is wordy and not idiomatic. The phrases was also an inspiration to and was significantly different are unnecessarily wordy. The phrase from that of is unclear and confusing.

14. (Book Question: 234)Though being tiny, blind, and translucent, a recently discovered species of catfish lessens their vulnerability with thickened bones and armor plates on their sides. A. Though being tiny, blind, and translucent, a recently discovered species of catfish lessens their vulnerability with thickened bones and armor plates on their sides. B. Though tiny, blind, and translucent, a recently discovered species of catfish has thickened bones and armor plates on its sides that lessen its vulnerability. C. A recently discovered species of catfish has thickened bones and armor plates on its sides that lessen their vulnerability, though tiny, blind, and translucent. D. Thickened bones and armor plates on their sides lessen the vulnerability of a recently discovered species of catfish that is tiny, blind, and translucent. E. Tiny, blind, and translucent, thickened bones and armor plates on its sides lessen the vulnerability of a recently discovered species of catfish.

Correct Answer: B Selected Answer: E Logical predication; Agreement The point of the sentence is to explain two sets of features of a catfish species, one that makes the species seem vulnerable, and the other that reduces its vulnerability. The sentence as written introduces the unnecessary participial being and incorrectly refers to the singular species with the plural possessive pronoun "their." A The participial being makes the sentence unnecessarily wordy. The plural pronoun their has no clear antecedent; it cannot refer to species, because the sentence has already established, with the singular verb lessens, that it is using species as a singular noun. Also, the sentence illogically says that the species (not its physical characteristics) lessens their vulnerability. B Correct. The opening set of adjectives introduced by the contrastive term though is efficient and sets up a contrast between the features that make the species vulnerable and those that make it less vulnerable. The singular pronoun its correctly refers to the singular noun species. C The referent for the plural possessive pronoun their is unclear; it is also unclear what the set of adjectives introduced by though is supposed to modify. The sentence structure suggests, nonsensically, that they modify armor plates. D This version of the sentence confusingly refers to species as both plural (their sides) and singular (that is). E The opening set of adjectives (tiny, blind, and translucent) nonsensically describes bones and plates.

72. (Book Question: 791)Found only in the Western Hemisphere and surviving through extremes of climate, hummingbirds' range extends from Alaska to Tierra del Fuego, from sea-level rain forests to the edges of Andean snowfields and ice fields at altitudes of 15,000 feet. A. Found only in the Western Hemisphere and surviving through extremes of climate, hummingbirds' range extends B. Found only in the Western Hemisphere, hummingbirds survive through extremes of climate, their range extending C. Hummingbirds, found only in the Western Hemisphere and surviving through extremes of climate, with their range extending D. Hummingbirds, found only in the Western Hemisphere and surviving through extremes of climate, their range extends E. Hummingbirds are found only in the Western Hemisphere, survive through extremes of climate, and their range extends

Correct Answer: B Selected Answer: E Logical predication; Grammatical construction This sentence makes three points about hummingbirds: they live in the Western Hemisphere, they survive extreme climates, and their range is wide and varied. Hummingbirds, not hummingbirds' range, should be the subject of the sentence. A The opening modifier, found . . . and surviving . . . should modify hummingbirds, not, as it does, hummingbirds' range. B Correct. Hummingbirds is the subject of the sentence, and the use of the absolute phrase their range extending appropriately connects the final clause to the rest of the sentence. C This version of the sentence has no main verb for the subject Hummingbirds and as a result is ungrammatical. D Like (C), this version of the sentence has no main verb for the subject Hummingbirds. E This version of the sentence begins with the subject Hummingbirds attached to a pair of verb phrases (are found . . . and survive . . .) followed by the conjunction and, which suggests that what follows should also be a verb phrase; instead, a new subject is introduced, their range. Replacing the comma before survive with and would render the sentence acceptable.

8. (Book Question: 230)Because the collagen fibers in skin line up in the direction of tension, surgical cuts made along these so-called Langer's lines sever fewer fibers and is less likely to leave an unsightly scar. A. Because the collagen fibers in skin line up in the direction of tension, surgical cuts made along these so-called Langer's lines sever fewer B. Because the collagen fibers in skin line up in the direction of tension, a surgical cut having been made along these so-called Langer's lines severs less C. Because the collagen fibers in skin line up in the direction of tension, a surgical cut made along these so-called Langer's lines severs fewer D. With the collagen fibers in skin lining up in the direction of tension, surgical cuts made along these so-called Langer's lines sever less E. With the collagen fibers in skin lining up in the direction of tension, a surgical cut made along these so-called Langer's lines sever fewer

Correct Answer: C Selected Answer: A Agreement; Diction This sentence explains a causal connection between the alignment of collagen fibers and the impact of a particular type of surgical cut. Because is appropriate to express that causal relationship. The singular verb in the phrase is less likely to leave requires a singular subject (cut) and must be coordinated with another singular verb (severs). Because fibers are countable, the correct modifier is fewer rather than less. A The plural subject cuts does not agree with the singular verb is. B The verb form having been made is inconsistent with the present tense verb severs; less inappropriately modifies countable fibers. C Correct. The adverbial conjunction because accurately captures the causal relationship expressed by the sentence. The singular subject cut agrees with the singular verbs severs and is, and fewer appropriately modifies countable fibers. D The preposition with does not capture the causal relationship expressed by the sentence; the plural subject cuts does not agree with the singular verbs (severs and is); and less is an inappropriate modifier for countable fibers. E As in (D), the preposition with fails to capture the causal relationship between alignment of fibers and scarring. The plural verb sever does not agree with the singular subject cut and the subsequent singular verb is.

67. (Book Question: 219)According to the Economic Development Corporation of Los Angeles County, if one were to count the Los Angeles metropolitan area as a separate nation, it would have the world's eleventh largest gross national product, that is bigger than that of Australia, Mexico, or the Netherlands. A. if one were to count the Los Angeles metropolitan area as a separate nation, it would have the world's eleventh largest gross national product, that is B. if the Los Angeles metropolitan area is counted as a separate nation, it has the world's eleventh largest gross national product, that being C. if the Los Angeles metropolitan area were a separate nation, it would have the world's eleventh largest gross national product, D. were the Los Angeles metropolitan area a separate nation, it will have the world's eleventh largest gross national product, which is E. when the Los Angeles metropolitan area is counted as a separate nation, it has the world's eleventh largest gross national product, thus

Correct Answer: C Selected Answer: A Diction; Verb form The point of this sentence is to explain the implications of a contrary-to-fact state of affairs (Los Angeles metropolitan area as a nation). The subjunctive verb form is needed (were . . . would have). To attribute this proposed state of affairs to the calculations of an anonymous agent (one) causes unnecessary wordiness. The implication of the hypothetical situation is that the Los Angeles area would have the eleventh-largest gross national product (GNP) in the world, a GNP that is further described as larger than the GNP of any of three nations named. This descriptive information is most efficiently presented as a terminal adjective phrase. A By introducing the subject one, the opening clause becomes unnecessarily wordy and indirect. The relative clause at the end of the sentence causes additional wordiness. The present indicative verb form is in the phrase that is bigger than . . . is inconsistent with the conditional context established earlier in the sentence (were . . . would). Since Los Angeles is not a nation, its national product is purely hypothetical and contrary to fact. B Because the if clause introduces a situation that is contrary to fact, the verbs is counted and has should be subjunctive and conditional, respectively (were counted and would have). The relative pronoun phrase that being is awkward, wordy, and repetitive. C Correct. The subjunctive mood of the verbs is appropriate to the contrary-to-fact situation being described, and the terminal adjective phrase without an introductory relative pronoun is an appropriate way of making the comparison among GNPs. D Although the opening subjunctive verb is appropriate, it must be followed by a conditional verb in the main clause; the relative clause at the end of the sentence, beginning with which is, is indirect and wordy. As in (A), is is not the most appropriate verb form to express a counterfactual condition. E The verbs is counted . . . has are incorrect for describing a contrary-to-fact situation. Beginning the final adjective phrase with the word thus makes the relationship of the phrase to the rest of the sentence unclear.

96. (Book Question: 767)The results of the company's cost-cutting measures are evident in its profits, which increased 5 percent during the first 3 months of this year after it fell over the last two years. A. which increased 5 percent during the first 3 months of this year after it fell B. which had increased 5 percent during the first 3 months of this year after it had fallen C. which have increased 5 percent during the first 3 months of this year after falling D. with a 5 percent increase during the first 3 months of this year after falling E. with a 5 percent increase during the first 3 months of this year after having fallen

Correct Answer: C Selected Answer: A Verb form; Agreement; Idiom This sentence describes two sequentially ordered indicators by which the results of a company's costcutting measures can be seen. The first indicator to be identified, a 5 percent increase in profits, occurred after the indicator mentioned next. The sentence therefore needs to clarify the sequence by presenting the first-identified indicator in a clause with a verb tense that indicates a later time period than the verb tense in the clause presenting the second-identified indicator. The subject of the relative clause, which, refers to the plural noun profits, so subsequent pronouns referring to these profits must also be plural. A The verb tenses do not distinguish between the times at which these indicators occurred; the singular it does not agree with the plural profits. B The verb tenses do not distinguish between the times when the indicators occurred; the singular it does not agree with the plural profits. C Correct. The verb tenses clearly indicate the sequence of events. D It is not clear what connection is being described by with; the prepositional phrase makes the sentence wordy and unclear. E It is not clear what connection is being described by with; the prepositional phrase makes the sentence wordy and unclear.

75. (Book Question: 288)Sound can travel through water for enormous distances, prevented from dissipating its acoustic energy as a result of boundaries in the ocean created by water layers of different temperatures and densities. A. prevented from dissipating its acoustic energy as a result of B. prevented from having its acoustic energy dissipated by C. its acoustic energy prevented from dissipating by D. its acoustic energy prevented from being dissipated as a result of E. preventing its acoustic energy from dissipating by

Correct Answer: C Selected Answer: E Logical predication; Rhetorical construction This sentence opens with a statement that sound can travel long distances through water and then explains why that is so: water layers in the ocean prevent acoustic energy from dissipating. Because dissipating is an intransitive verb, acoustic energy cannot be its object. A Dissipating is not a transitive verb, so acoustic energy cannot function as its object. B This version of the sentence is wordy, awkward, and indirect; from having . . . by erroneously suggests that the boundaries in the ocean are attempting to dissipate sound energy. C Correct. Here, acoustic energy is effectively modified by the participial prevented from dissipating. . . . D This version of the sentence is wordy, awkward, and indirect; being dissipated as a result of makes it unclear whether the boundaries contribute to energy loss or prevent it. E This version of the sentence nonsensically explains that sound prevents the dissipation of its own energy. A,B and E are worng as they mean that the sound is preventing its coustic energy from dissipating, however it is the boundries that are preventing as in C and D. D is rejected for using "being". So IMO C.

12. (Book Question: 769)Thelonious Monk, who was a jazz pianist and composer, produced a body of work both rooted in the stride-piano tradition of Willie (The Lion) Smith and Duke Ellington, yet in many ways he stood apart from the mainstream jazz repertory. A. Thelonious Monk, who was a jazz pianist and composer, produced a body of work both rooted B. Thelonious Monk, the jazz pianist and composer, produced a body of work that was rooted both C. Jazz pianist and composer Thelonious Monk, who produced a body of work rooted D. Jazz pianist and composer Thelonious Monk produced a body of work that was rooted E. Jazz pianist and composer Thelonious Monk produced a body of work rooted both

Correct Answer: D Selected Answer: B Grammatical construction; Rhetorical construction The subject of the sentence is Thelonious Monk, and the sentence tells about two things that he did: produced and stood apart. The work he produced was rooted in the mainstream (stride piano) jazz tradition, yet at the same time, he deviated from this tradition. The use of a relative clause (who was a jazz pianist . . .) or an appositive (the jazz pianist . . .) introduces unnecessary wordiness and grammatical complexity. Since only one point is being made about Monk's body of work, the appearance of the word both in the clause presenting the claim about Monk's work is deceptive as well as grammatically incorrect. A The relative clause introduces wordiness and confusion. B The appositive introduces wordiness and unnecessary grammatical complexity. C The sentence is a fragment because the main subject, Thelonious Monk, has no verb. D Correct. The sentence concisely identifies Thelonious Monk and expresses the single point about his work without unnecessary or misleading words. E The appearance of both is misleading, since only one point is being made about where Monk's musical roots are located. Main problem of this sentence is PARALLELISM: idiom: both X and Y --> X and Y must be parallel. Thelonious Monk, who was a jazz pianist and composer, produced a body of work both rooted in the stride-piano tradition of Willie (The Lion) Smith and Duke Ellington, yet in many ways he stood apart from the mainstream jazz repertory. A. Thelonious Monk, who was a jazz pianist and composer, produced a body of work both rooted Wrong. Not parallel: both rooted in the stride-piano tradition of Willie Smith and Duke Ellington B. Thelonious Monk, the jazz pianist and composer, produced a body of work that was rooted both Wrong. Same as in A. rooted both in the stride-piano tradition of Willie Smith and Duke Ellington C. Jazz pianist and composer Thelonious Monk, who produced a body of work rooted Wrong. A clause "who produced a body......" is MODIFIER ==> It means if you eliminate it, the sentence is still sensible. Let try: Jazz pianist and composer Thelonious Monk, who produced a body of work rooted in the stride-piano tradition of Willie (The Lion) Smith and Duke Ellington, yet in many ways he stood apart from the mainstream jazz repertory.. ==> The red part is not a sentence. --> Wrong. The green part is a complete sentence with S and V. D. Jazz pianist and composer Thelonious Monk produced a body of work that was rooted Correct. E. Jazz pianist and composer Thelonious Monk produced a body of work rooted both Wrong. Same problem as in A and B. rooted both in the stride-piano tradition of Willie Smith and Duke Ellington Hope it helps.

74. (Book Question: 270)Until 1868 and Disraeli, Great Britain had no prime ministers not coming from a landed family. A. Until 1868 and Disraeli, Great Britain had no prime ministers not coming B. Until 1868 and Disraeli, Great Britain had had no prime ministers who have not come C. Until Disraeli in 1868, there were no prime ministers in Great Britain who have not come D. It was not until 1868 that Great Britain had a prime minister—Disraeli—who did not come E. It was only in 1868 and Disraeli that Great Britain had one of its prime ministers not coming

Correct Answer: D Selected Answer: C Verb form; Idiom This sentence explains how Disraeli marked a turning point in British history: he was the first prime minister who did not come from the landed gentry. The placement of the double negative is crucial. While no prime ministers not coming from . . . is hard to untangle, [n]ot until . . . that Great Britain had a prime minister who did not come . . . separates the negatives into separate clauses, making them easier to decode. An appropriate way to express the temporal relationship is to use the idiomatic phrase not until . . . that. A The phrase no prime ministers not coming is unnecessarily confusing. It is also confusing to follow the preposition until with two very different types of objects—a date and a person. B As in (A), the double negative and unlike objects of the prepositional phrase starting with until are confusing. Additionally, the verb form have not come, which is the present-perfect tense, is inappropriate following the past perfect had had in this context. C The present-perfect tense (have not come) is inappropriate after the past tense were in this context. Until Disraeli is imprecise and incomplete. Before Disraeli's term in 1868 . . . or Until Disraeli became prime minster in 1868 would work. D Correct. This version correctly uses the idiomatic construction not until . . . that, and it correctly uses past tense throughout. E While it makes sense to say that a historical change occurred in 1868, it does not make sense to say that it occurred in Disraeli. It is unidiomatic to say had one of its prime ministers not coming.

Inuits of the Bering Sea were in isolation from contact with Europeans longer than Aleuts or Inuits of the North Pacific and northern Alaska. A. in isolation from contact with Europeans longer than B. isolated from contact with Europeans longer than C. in isolation from contact with Europeans longer than were D. isolated from contact with Europeans longer than were E. in isolation and without contacts with Europeans longer than

Correct Answer: D Selected Answer: E Idiom; Logical predication The construction in isolation from is awkward; the idiomatic way to express this idea is isolated from. The comparison is ambiguous; it could mean the Bering Sea Inuits were isolated from Europeans longer than they were isolated from Aleuts and other Inuits or that they were isolated from Europeans longer than Aleuts and other Inuits were isolated from Europeans. Adding were after than will solve this problem. A In isolation from is not the correct idiom. The comparison is ambiguous. B The comparison is ambiguous. C In isolation from is not the correct idiom. D Correct. The idiom isolated from is correctly used in this sentence. The comparison is clear and unambiguous. E In isolation . . . without is incorrect and confusing. The comparison is ambiguous.

30. (Book Question: 802)A ruined structure found at Aqaba, Jordan, was probably a church, as indicated in its eastward orientation and by its overall plan, as well as artifacts, such as glass oil-lamp fragments, found at the site. A. A ruined structure found at Aqaba, Jordan, was probably a church, as indicated in its eastward orientation and by its overall plan, as well as B. A ruined structure found at Aqaba, Jordan, once probably being a church, was indicated by its eastward orientation, overall plan, and C. Indicating that a ruined structure found at Aqaba, Jordan, was probably a church were its eastward orientation and overall plan, but also the D. A ruined structure found at Aqaba, Jordan, was probably a church, as indicates its eastward orientation and overall plan, as well as the E. That a ruined structure found at Aqaba, Jordan, was probably a church is indicated by its eastward orientation and overall plan, as well as by the

Correct Answer: E Selected Answer: A Logical predication; Parallelism This sentence explains why a currently ruined structure probably used to be a church. In the best-worded answer choice, the abstract subject (the probability that a certain hypothesis is true) is explained abstractly in a relative clause (That a ruined structure was probably . . .) followed by a passive verb (is indicated), followed by the prepositional phrase (by . . .), which is completed by a parallel listing of forms of evidence, all presented as noun phrases in the expression, (by) (its) A + B, as well as (by) C. A This version makes the relationship between as well as . . . and the rest of the sentence unclear. The most plausible hypothesis is that the artifacts are another of the types of evidence, parallel with the structure's orientation and its plan. The sentence violates the parallelism required in the list by failing to supply the preposition in the final item following as well as. B The subject of this version of the sentence, structure, is completed by the verb was indicated, creating the illogical assertion that the structure itself, rather than its probable identity, was indicated by its orientation, plan, and attendant artifacts. C Opening the sentence with the predicate adjective indicating leads to a confusing and awkward withholding of the subjects orientation . . . plan . . . artifacts. D The only plausible subject for the singular verb indicates is the plural orientation . . . plan, as well as the artifacts. Very little about this sentence makes sense. E Correct. The relationships among the parts of the sentence are clear and logical.

38. (Book Question: 723)Emily Dickinson's letters to Susan Huntington Dickinson were written over a period beginning a few years before Susan's marriage to Emily's brother and ending shortly before Emily's death in 1886, outnumbering her letters to anyone else. A. Dickinson were written over a period beginning a few years before Susan's marriage to Emily's brother and ending shortly before Emily's death in 1886, outnumbering B. Dickinson were written over a period that begins a few years before Susan's marriage to Emily's brother and ended shortly before Emily's death in 1886, outnumber C. Dickinson, written over a period beginning a few years before Susan's marriage to Emily's brother and that ends shortly before Emily's death in 1886 and outnumbering D. Dickinson, which were written over a period beginning a few years before Susan's marriage to Emily's brother, ending shortly before Emily's death in 1886, and outnumbering E. Dickinson, which were written over a period beginning a few years before Susan's marriage to Emily's brother and ending shortly before Emily's death in 1886, outnumber

Correct Answer: E Selected Answer: A Parallelism; Grammatical construction The main point of the sentence is that Dickinson's letters to her sister-in-law outnumber her letters to anyone else. To emphasize this point, outnumber should be the main verb, and the description introduced by the passive verb were written needs to be changed from a main clause to an adjectival phrase. A The long, wordy opening clause gives too much emphasis to the period when Dickinson's letters were written; it is unclear what outnumbering refers to. B The verbs describing the letter-writing period (begins and ended) are not parallel. C The verbs describing the letter-writing period need to be in parallel form and agree in tense—e.g., beginning and ending or that began and that ended; this is a fragment because it lacks a main verb for letters. D The lack of a main verb for the subject of the sentence, letters, makes this a fragment. E Correct. The information about the period when Dickinson's letters were written is contained in an adjectival phrase set off by commas, and the main verb outnumber refers clearly to letters.

41. (Book Question: 723)Emily Dickinson's letters to Susan Huntington Dickinson were written over a period beginning a few years before Susan's marriage to Emily's brother and ending shortly before Emily's death in 1886, outnumbering her letters to anyone else. A. Dickinson were written over a period beginning a few years before Susan's marriage to Emily's brother and ending shortly before Emily's death in 1886, outnumbering B. Dickinson were written over a period that begins a few years before Susan's marriage to Emily's brother and ended shortly before Emily's death in 1886, outnumber C. Dickinson, written over a period beginning a few years before Susan's marriage to Emily's brother and that ends shortly before Emily's death in 1886 and outnumbering D. Dickinson, which were written over a period beginning a few years before Susan's marriage to Emily's brother, ending shortly before Emily's death in 1886, and outnumbering E. Dickinson, which were written over a period beginning a few years before Susan's marriage to Emily's brother and ending shortly before Emily's death in 1886, outnumber

Correct Answer: E Selected Answer: A Parallelism; Grammatical construction The main point of the sentence is that Dickinson's letters to her sister-in-law outnumber her letters to anyone else. To emphasize this point, outnumber should be the main verb, and the description introduced by the passive verb were written needs to be changed from a main clause to an adjectival phrase. A The long, wordy opening clause gives too much emphasis to the period when Dickinson's letters were written; it is unclear what outnumbering refers to. B The verbs describing the letter-writing period (begins and ended) are not parallel. C The verbs describing the letter-writing period need to be in parallel form and agree in tense—e.g., beginning and ending or that began and that ended; this is a fragment because it lacks a main verb for letters. D The lack of a main verb for the subject of the sentence, letters, makes this a fragment. E Correct. The information about the period when Dickinson's letters were written is contained in an adjectival phrase set off by commas, and the main verb outnumber refers clearly to letters.

11. (Book Question: 277)Downzoning, zoning that typically results in the reduction of housing density, allows for more open space in areas where little water or services exist. A. little water or services exist B. little water or services exists C. few services and little water exists D. there is little water or services available E. there are few services and little available water

Correct Answer: E Selected Answer: C Diction; Agreement In this sentence, the adjective little correctly modifies the noun water because water is not a countable quantity. However, the noun services is a countable quantity and must be modified by few, not by little. Logically, the areas described would suffer from both little water and few services at the same time, so the correct conjunction is and, not or. This compound subject requires a plural verb. A Services should be modified by few, not little. B The singular verb exists does not agree with the plural subject services. When a compound subject is joined by or, the verb agrees with the closer subject. C When a compound subject consists of two distinct units joined by the conjunction and, the verb must be plural. D Little cannot modify services. E Correct. In this sentence, few correctly modifies services; and correctly joins services and water.

47. (Book Question: 796)Most efforts to combat such mosquito-borne diseases like malaria and dengue have focused either on the vaccination of humans or on exterminating mosquitoes with pesticides. A. like malaria and dengue have focused either on the vaccination of humans or on exterminating B. like malaria and dengue have focused either on vaccinating of humans or on the extermination of C. as malaria and dengue have focused on either vaccinating humans or on exterminating D. as malaria and dengue have focused on either vaccinating of humans or on extermination of E. as malaria and dengue have focused on either vaccinating humans or exterminating

Correct Answer: E Selected Answer: C Diction; Parallelism The phrase such . . . diseases like malaria and dengue is not a correct way in English to indicate that the two diseases mentioned are examples of a larger category; the correct expression is such . . . as. . . . A This use of such . . . like . . . is incorrect English; the correct expression is such . . . as.. . . It is better to keep the preposition on close to the verb it goes with, focus, so as not to repeat it. B The correct expression is such . . . as . . . . It is better to keep the preposition on close to the verb it goes with, focus, so as not to repeat it. This use of the gerund vaccinating (followed by of) would normally be preceded by the, but this would make the phrase awkward. It would be preferable to use vaccination, which is parallel to extermination. C This answer choice incorrectly repeats the preposition on before exterminating. D This answer choice incorrectly repeats the preposition on before extermination. This use of the gerund vaccinating (followed by of) would normally be preceded by the, but this would make the phrase awkward. It would be preferable to use vaccination, which is parallel to extermination. E Correct. This version uses either . . . or . . . correctly and appropriately uses the parallel forms vaccinating and exterminating.

42. (Book Question: 193)Global warming is said to be responsible for extreme weather changes, which, like the heavy rains that caused more than $2 billion in damages and led to flooding throughout the state of California, and the heat wave in the northeastern and midwestern United States, which was also the cause of a great amount of damage and destruction. A. which, like the heavy rains that caused more than $2 billion in damages and led to flooding throughout the state of California, B. which, like the heavy rains that throughout the state of California caused more than $2 billion in damages and led to flooding, C. like the heavy flooding that, because of rains throughout the state of California, caused more than $2 billion in damages, D. such as the heavy flooding that led to rains throughout the state of California causing more than $2 billion in damages, E. such as the heavy rains that led to flooding throughout the state of California, causing more than $2 billion in damages,

Correct Answer: E Selected Answer: C Grammatical construction: Logical predication This sentence introduces the claim that global warming is considered to be the cause of extreme weather changes and then illustrates these changes with two examples introduced by the phrase such as. The correct causal sequence of events in the first example is heavy rain, which caused significant damage and flooding. The relative pronoun which, referring to changes, is lacking a verb to complete the relative clause. A The relative pronoun, which, is without a verb. The phrase which, like the heavy rains incorrectly suggests that the extreme weather is something different from the heavy rains and the heat wave, and that each of these three phenomena separately caused damage and destruction. B As in (A), the relative pronoun has no verb to complete the phrase, and the sequence of events in the first example positions costly destruction and flooding as two separate or unrelated results of the rain. C The causal sequence in the first example is confusing, suggesting, somewhat implausibly, that heavy flooding occurred on its own but caused damage only because of the rain. D The sequential logic of the first example is confused—indicating, implausibly, that flooding caused heavy rain and that the rain, but not the flooding, caused more than $2 billion in damages. E Correct. This version of the sentence correctly uses the phrase such as to introduce the two examples of extreme weather changes, and it correctly identifies the sequence of events in the first example.

140. (Book Question: 795)Australian embryologists have found evidence that suggests that the elephant is descended from an aquatic animal, and its trunk originally evolving as a kind of snorkel. A. that suggests that the elephant is descended from an aquatic animal, and its trunk originally evolving B. that has suggested the elephant descended from an aquatic animal, its trunk originally evolving C. suggesting that the elephant had descended from an aquatic animal with its trunk originally evolved D. to suggest that the elephant had descended from an aquatic animal and its trunk originally evolved E. to suggest that the elephant is descended from an aquatic animal and that its trunk originally evolved

Correct Answer: E Selected Answer: C Parallelism; Verb form The clearest, most economical way of expressing the two things suggested by Australian embryologists' evidence is to format them as relative clauses serving as parallel direct objects of the verb suggest. It is awkward and confusing to string together relative clauses: evidence that suggests that the elephant. . . . A clearer way of making this connection is to turn the verb suggests into a participle modifying evidence. The word descended is a predicate adjective following the present-tense verb is and describing the present-day elephant. The verb evolved should be past tense because it describes how the trunk of the elephant originally evolved, not how it is evolving today. A The string of relative phrases is awkward and confusing; the phrase following the conjunction and is not parallel with the relative clause that the elephant is descended. . . . B The evidence still suggests these things about the evolution of the elephant and its trunk, so the present-perfect verb tense is inaccurate. C Had descended is the wrong verb tense; with cannot be followed by an independent clause. D Had descended is the wrong tense; the phrase following the conjunction and does not parallel the relative clause that precedes the conjunction. E Correct. The two dependent clauses beginning with that are in parallel form and contain verbs in the correct tenses.

81. (Book Question: 286)Sixty-five million years ago, according to some scientists, an asteroid bigger than Mount Everest slammed into North America, which, causing plant and animal extinctions, marks the end of the geologic era known as the Cretaceous Period. A. which, causing plant and animal extinctions, marks B. which caused the plant and animal extinctions and marks C. and causing plant and animal extinctions that mark D. an event that caused plant and animal extinctions, which marks E. an event that caused the plant and animal extinctions that mark

Correct Answer: E Selected Answer: D Logical predication; Agreement This sentence describes a two-part sequence of events, the second of which has led to a particular categorization of geological time. In order to clarify that it is not the first event (asteroid strike) that produced the time division (end of the Cretaceous Period), but the first event's consequences (biological extinctions), the sentence needs an appositive form to restate the content of the main clause (an event), followed by a two-part chain of relative clauses (that caused . . . that mark . . .). A The antecedent for the relative pronoun which is ambiguous; it is therefore unclear what marks the end of the Cretaceous Period. B The antecedent of which is unclear; the compound verbs caused and marks fail to indicate that the extinctions, not the asteroid strike, are significant markers of geological time. C Following the conjunction and with a participial rather than a main verb is grammatically incorrect because it violates parallelism and produces a fragment at the end of the sentence. D Which, referring to extinctions, should be followed by a plural verb. E Correct. The sentence is unambiguous, and the verbs agree with their subjects.

51. (Book Question: 299)Once numbering in the millions worldwide, it is estimated that the wolf has declined to 200,000 in 57 countries, some 11,000 of them to be found in the lower 48 United States and Alaska. A. it is estimated that the wolf has declined to 200,000 in 57 countries, some B. the wolf is estimated to have declined to 200,000 in 57 countries, with approximately C. the wolf has declined to an estimate of 200,000 in 57 countries, some D. wolves have declined to an estimate of 200,000 in 57 countries, with approximately E. wolves have declined to an estimated 200,000 in 57 countries, some

Correct Answer: E Selected Answer: D Logical predication; Idiom The predicate numbered must have its logically correct subject, which is wolves. Although the wolf can be used to refer collectively to wolves as a category, the noun should be plural in this case since the sentence refers to numbers of them and since agreement is needed between the noun and the plural pronoun them. Given the plural subject, the verb in the independent clause should be have declined. The object of decline to should be a word or phrase naming a number or estimated number (e.g., a phrase such as an estimated 200,000), not the phrase an estimate. A The subject of the main clause (it) seems to supply the subject of numbering, so the latter does not have its correct logical subject, which should be a word or phrase referring to wolves. B The wolf cannot correctly be taken as the subject of numbering, as it is singular and in disagreement with them occurring later in the sentence. C The wolf cannot correctly be taken as the subject of numbering, as it is singular and disagrees with the later them; also, a decline is strictly speaking to a number, not to an estimate. D Although wolves is a correct subject for numbering, a decline should be to a number, not to an estimate. E Correct. Wolves is a proper subject for numbering and agrees with the later them. The decline is correctly said to be to a number, an estimated 200,000.

27. (Book Question: 218)In recent years cattle breeders have increasingly used crossbreeding, in part that their steers should acquire certain characteristics and partly because crossbreeding is said to provide hybrid vigor. A. in part that their steers should acquire certain characteristics B. in part for the acquisition of certain characteristics in their steers C. partly because of their steers acquiring certain characteristics D. partly because certain characteristics should be acquired by their steers E. partly to acquire certain characteristics in their steers

Correct Answer: E Selected Answer: D Parallelism; Rhetorical construction The sentence gives two reasons that cattle breeders use crossbreeding; these reasons should be introduced in parallel ways with the word partly. The infinitive to acquire clearly and concisely conveys the purpose of the crossbreeding. A In part should be partly. Use of the relative clause that their steers should acquire . . . is ungrammatical. B In part should be partly. Use of prepositional phrases is wordy and awkward. C Because of suggests that crossbreeding has occurred because the steers have already acquired certain characteristics. D Passive voice should be acquired by is awkward and illogical. E Correct. In this sentence, the word partly is used to introduce both reasons; the phrase to acquire certain characteristics is clear and concise.

Under high pressure and intense heat, graphite, the most stable form of pure carbon, changes into the substance commonly referred to as diamond and remaining this way whether or not the heat and pressure are removed. A. remaining this way whether or not B. remaining like that even as C. remaining as such whether or not D. remains in this way although E. remains thus even when

Correct Answer: E Selected Answer: D Parallelism; Rhetorical construction This sentence tells of two things that happen to graphite under intense heat and pressure, and these are best presented as parallel predicates—changes and remains. Thus is the most economical way to say this way, like that, as such, or in this way. A Remaining should be a main verb, parallel with changes. B Remaining should be parallel with the other main verb, changes; even as suggests the meaning of while, which is not the intent of the sentence. C Remaining should be parallel with changes; whether or not is unnecessarily wordy. D In this way is unnecessarily wordy; although indicates that the heat and pressure are always or definitely removed, but this makes little sense in relation to the rest of the sentence. E Correct. The sentence coherently refers to the possibility of heat and pressure being removed. The sentence is clear and concise and properly uses parallel verb forms.

99. (Book Question: 251)Twenty-two feet long and 10 feet in diameter, the AM-1 is one of the many new satellites that is a part of 15 years effort of subjecting the interactions of Earth's atmosphere, oceans, and land surfaces to detailed scrutiny from space. A. satellites that is a part of 15 years effort of subjecting the interactions of Earth's atmosphere, oceans, and land surfaces B. satellites, which is a part of a 15-year effort to subject how Earth's atmosphere, oceans, and land surfaces interact C. satellites, part of 15 years effort of subjecting how Earth's atmosphere, oceans, and land surfaces are interacting D. satellites that are part of an effort for 15 years that has subjected the interactions of Earth's atmosphere, oceans, and land surfaces E. satellites that are part of a 15-year effort to subject the interactions of Earth's atmosphere, oceans, and land surfaces

Correct Answer: E Selected Answer: E Rhetorical construction; Logical predication This sentence describes one satellite and identifies it as part of a larger space project designed to scrutinize Earth's ocean, land, and atmospheric interactions. The relative pronoun that refers to satellites, so it should be followed by a plural verb. The idiomatic expression is effort to rather than effort of. The correct adjectival term is 15-year rather than 15 years. A The relative pronoun that refers to satellites, so it should be followed by the plural verb are; effort to is the correct idiomatic expression; as an adjective, 15 years becomes 15-year. B In this version of the sentence, it is unclear what the relative pronoun which refers to—if it refers to satellites, it should be followed by a plural verb. Presenting the object of the verb subject as a phrase beginning with how and ending with the verb interact produces a sentence that seems to be about how various conditions react to detailed scrutiny from space. C This sentence too seems to be making a nonsensical statement about how conditions are interacting to detailed scrutiny. It is not clear whether part refers to satellites or the AM-1. D This version is wordy and confusing because of the sequence of relative clauses beginning with that. E Correct. The sentence is clearly worded and logically coherent.

The new image of Stone Age people as systematic hunters of large animals, rather than merely scavenging for meat, have emerged from the examination of tools found in Germany, including three wooden spears that archaeologists believe to be above 400,000 years old. A. merely scavenging for meat, have emerged from the examination from the examination of tools found in Germany, including B. as mere scavenging for meat, have emerged from examining tools found in Germany, which include C. as mere meat scavengers, has emerged from examining tools found in Germany that includes D. mere scavengers of meat, has emerged from the examination of tools found in Germany, which includes E. mere scavengers of meat, has emerged from the examination of tools found in Germany, including

E,如果d的includes换成include就是对的,in germany不能省略,所以是可以跳过去修饰,但是tools要用复数 including用的特别不好,忍了 Answer to your first question: Yes, it is alright to not repeat "as" after "rather than" because it is implied or understood. "As" already appears once in the sentence. Another thing that we need to note in Choice C is parallelism. In the parallel list, the first entity is "hunters of large animals" and the second one is "meat scavengers". Now, it is not always necessary for the entities in the parallel list to be absolutely parallel. However, in this case, it is possible. We can write "meat scavengers" as "scavengers of meat" that will make the entities absolutely parallel. So go for it. Answer to your second question: Yes, "examining tools" have a little problem. Here "examining" is now an adjective that is modifying "tools", suggesting that the "tools" are used for examining things. It no longer conveys that the new image has emerged from the examination of the tool. Let's take these sentences: The smile of the baby is beautiful. (smile is beautiful) The smiling baby is beautiful. (baby is beautiful) So be careful of the change in the words in the original choice. They might change the meaning of the sentence.

47. (Book Question: 753)The Olympic Games helped to keep peace among the pugnacious states of the Greek world in that a sacred truce was proclaimed during the festival's month. A. world in that a sacred truce was proclaimed during the festival's month B. world, proclaiming a sacred truce during the festival's month C. world when they proclaimed a sacred truce for the festival month D. world, for a sacred truce was proclaimed during the month of the festival E. world by proclamation of a sacred truce that was for the month of the festival

F For A And N Nor B But O Or Y Yet S So Correct Answer: D Selected Answer: D Idiom; Rhetorical construction This sentence depends on using the correct conjunction to join two independent clauses. In that is a conjunction that means inasmuch as; because in that has largely gone out of use, it is considered stilted and overly formal. It also uses two words when one would do. In this sentence, the second clause explains the first one, so the conjunction for, meaning because, is the most appropriate choice for joining the two independent clauses of the compound sentence. Festival's month is an awkward and imprecise use of the possessive; during the month of the festival is clearer. A In that is stilted and overly formal. B It is not clear who would be doing the proclaiming; a clause is preferable to a phrase here. C They is ambiguous, possibly referring to either the states or the Games. The phrase truce for the festival month loses the sense that it's to take place for the duration of the month. D Correct. In this sentence, the conjunction for joins the two clauses correctly and economically. E Wordy and awkward construction.

句子改错 If Abraham Lincoln were born in Livonia, he cannot become the President of the United States.

If Abraham Lincoln had been born in Livonia, he could not have become the President of the United States.

110. An attempt to ratify the Equal Rights Amendment, begun almost two decades ago, has been unsuccessful despite efforts by many important groups, including the National Organization for Women. (A) to ratify the Equal Rights Amendment, begun almost two decades ago, (B) begun almost two decades ago, for ratifying the Equal Rights Amendment (C) begun for ratifying the Equal Rights Amendment almost two decades ago (D) at ratifying the Equal Rights Amendment, begun almost two decades ago, (E) that has begun almost two decades ago to ratify the Equal Rights Amendment

In E that has begun is wrong.Whenever a definite time is specified (like 'two decades ago' in this case) you have to use simplepast tense A modifier may in some exceptional cases refer to a noun slightly far away from it. One such case is when another "mission critical" modifier comes in between the modifier and the noun it refers to. A "mission critical" modifier is frequently a prepositional phrase that cannot be placed in another position in the sentence conveniently. In this case as well, the mission critical modifier "to ratify the Equal Rights Amendment" comes in between the modifier "begun almost two decades ago" and the noun it refers to ("attempt"). Such usage is acceptable in GMAT. A

67. (Book Question: 684)Like ancient Egyptian architectural materials that were recycled in the construction of ancient Greek Alexandria, so ancient Greek materials from the construction of that city were reused in subsequent centuries by Roman, Muslim, and modern builders. A. Like ancient Egyptian architectural materials that were recycled in the construction of B. Like recycling ancient Egyptian architectural materials to construct C. Just as ancient Egyptian architectural materials were recycled in the construction of D. Just as they recycled ancient Egyptian architectural materials in constructing E. Just like ancient Egyptian architectural materials that were recycled in constructing

Just like so vs just as so. 应该用just as...so Correct Answer: C Selected Answer: E Diction; Parallelism When two situations are asserted to be similar, the proper way to express this is with the paired expressions just as . . . so . . ., not like . . . so. Moreover, the two compared situations should be expressed as clauses, not as noun phrases. Thus the clause ancient Egyptian architectural materials were recycled . . . is correct, as opposed to a noun phrase like ancient Egyptian architectural materials that were recycled . . . A Just as and a following clause with a passive verb are preferred, but instead this option has like and a following noun phrase (ancient Egyptian materials . . .). It appears, somewhat implausibly, to say that the ancient Greek materials were similar to the earlier ancient Egyptian ones in that both were used by Roman, Muslim, and modern builders. B Just as and a following clause with a passive verb are preferred, but instead this option has like and a following noun phrase (recycling ancient Egyptian materials . . .). It appears illogically to say that the action of recycling was similar to the ancient Greek materials in that both were reused in subsequent centuries. C Correct. The expressions just as and so are paired to link the two clauses in parallel, and both clauses use the passive construction. D Just as is used to connect two clauses, which is good, but the first clause employs the active construction rather than the preferred passive, so there is a failure of parallelism. Also, it is unclear what they refers to. The sentence appears to say illogically that some unidentified group's action of recycling was similar to the ancient Greek materials' being reused. E Just as introducing a clause with a passive verb is preferable, but this sentence uses like and a following noun phrase (ancient Egyptian materials . . .). It appears, somewhat implausibly, to say that the ancient Greek materials were similar to the earlier ancient Egyptian ones in that both were used by Roman, Muslim, and modern builders.

Today's technology allows manufacturers to make small cars more fuel-efficient now than at any time in their production history. (A) small cars more fuel-efficient now than at any time in their (B) small cars that are more fuel-efficient than they were at any time in their (C) small cars that are more fuel-efficient than those at any other time in (D) more fuel-efficient small cars than those at any other time in their (E) more fuel-efficient small cars now than at any time in what is the difference between B and C,plz explain

Let's analyze original sentence and all the answer choices one by one. There are two mistakes in original sentence. 1. For the idiom more x than y, X and Y should be parallel. Here X = fuel-efficient and Y = at any time in their production history(lets say 1990). Which are not parallel. Consider right usage ex : My car is more fuel efficient than your car. OR My car is more fuel efficient than powerful. 2. Usage of pronoun 'Their". Their can refer to both "cars" and "manufactures" so there is no clear antecedent for "their". Now see the answer choices. A is wrong because of above reasons. B is wrong because : 1. Usage of "they" and " Their" C is still has pronoun issue but meaning wise it is best . It is comparing small cars with those old cars. Lets have same substitution Today's technology allows manufacturers to make small cars that are more fuel-efficient than those at any other time in production history. Today's technology allows manufacturers to make small cars that are more fuel-efficient than those at 1990. D We can prefer C over D also for modifier reason : D has two continuous modifier "more fuel-efficient" and "small" intended to modify cars. In c we don't have continuous modifier, that is better. Usage of "their" again leading to ambiguity along with "those" E. It again has 2 modifier continuously.

More than 300 rivers drain into Siberia's Lake Baikal, which holds 20 percent of the world's fresh water, more than all the North American Great Lakes combined. (A) More than 300 rivers drain into Siberia's Lake Baikal, which holds 20 percent of the world's fresh water, more than all the North American Great Lakes combined. (B) With 20 percent of the world's fresh water, that is more than all the North American Great Lakes combined, Siberia's Lake Baikal has more than 300 rivers that drain into it. (C) Siberia's Lake Baikal, with more than 300 rivers draining into it, it holds more of the world's fresh water than all that of the North American Great Lakes combined, 20 percent. (D) While more than 300 rivers drain into it, Siberia's lake Baikal holds 20 percent of the world's fresh water, which is more than all the North American Great Lakes combined. (E) More than all the North American Great Lakes combined, Siberia's Lake Baikal, with more than 300 rivers draining into it, holds 20 percent of the world's fresh water.

Modifier rules http://magoosh.com/gmat/2013/modifiers-on-the-gmat-sentence-correction/ (C) Siberia's Lake Baikal(, with more than 300 rivers draining into it,) it holds more of the world's fresh water than all that of the North American Great Lakes combined, 20 percent. Option C is something like: "Siberia's Lake Baikal IT holds". The pronoun is not correct. Aslo "20 percent" is misplaced. The problem with D is "while": it means -at the same time - or shows a contrast => in both cases it not correct in the sentence (none of the two things happens here). (D) While more than 300 rivers drain into it, Siberia's lake Baikal holds 20 percent of the world's fresh water, which is more than all the North American Great Lakes combined. (E) More than all the North American Great Lakes combined (initial modifier), Siberia's Lake Baikal, with more than 300 rivers draining into it, holds 20 percent of the world's fresh water. E says that "Siberia's Lake Baikal is more than all the North American Great Lakes combined" => misplaced modifier (B) With 20 percent of the world's fresh water, that is more than all the North American Great Lakes combined, Siberia's Lake Baikal has more than 300 rivers that drain into it. B says that "20% of the world's fresh water is more than all US lakes" A is correct

Recently documented examples of neurogenesis, the production of new brain cells, include the brain growing in mice when placed in a stimulating environment or neurons increasing in canaries that learn new songs. A. the brain growing in mice when placed in a stimulating environment or neurons increasing in canaries that B. mice whose brains grow when they are placed in a stimulating environment or canaries whose neurons increase when they C. mice's brains that grow when they are placed in a stimulating environment or canaries' neurons that increase when they D. the brain growth in mice when placed in a stimulating environment or the increase in canaries' neurons when they E. brain growth in mice that are placed in a stimulating environment or an increase in neurons in canaries that

OA is E! Interesting that nobody got it right :? My answer was B when I first saw this SC. This is what happens (according to me): it parallels examples of the production of new brain cells, i.e. nouns or noun phrases. However, the trick is the modifiers following these noun/noun phrases. The modifiers are either subordinate clauses or relative clauses. One should read the options with the modifying clauses in mind and correct accordingly. A. the brain growing in mice when placed in a stimulating environment or neurons increasing in canaries that - the modifying clauses are not parallel. B. mice whose brains grow when they are placed in a stimulating environment or canaries whose neurons increase when they - incorrect use of mice and canaries as examples (examples need to be of the production of new brain cells). Also, ambiguous use of they C. mice’s brain that grow when they are placed in a stimulating environment or canaries’ neurons that increase when they - incorrect use of mice's brain and canaries' neurons as examples. Also, ambiguous use of they in the second example D. the brain growth in mice when placed in a stimulating environment or the increase in canaries’ neurons when they - the examples are not parallel (canaries' neurons is in possessive form whereas the brain growth is not). Also, the modifying clauses are ambigious. E.g., they in the second example E. brain grown in mice that are placed in a stimulating environment or increase in neurons in canaries that - the examples and the modifying clauses are parallel.

Only seven people this century have been killed by the great white shark, the man-eater of the movies—less than those killed by bee stings. A. movies—less than those B. movies—fewer than have been C. movies, which is less than those D. movies, a number lower than the people E. movies, fewer than the ones

Only seven people this century have been killed by the great white shark, the man-eater of the movies—less than those killed by bee stings. A. movies—less than those people are countable, less is used for non-countable things - INCORRECT!! B. movies—fewer than have been CORRECT!!! C. movies, which is less than thoseless should not be used for countable things - INCORRECT!! D. movies, a number lower than the peoplenumber should not be compared with people - INCORRECT!! E. movies, fewer than the onescomma makes this modifier, fewer than the pones seems to modify movies - INCORRECT!! IMO, answer is B

62. (Book Question: 793)Scientists have recently found evidence that black holes—regions of space in which matter is so concentrated and the pull of gravity so powerful that nothing, not even light, can emerge from them—probably exist at the core of nearly all galaxies and the mass of each black hole is proportional to its host galaxy. A. exist at the core of nearly all galaxies and the mass of each black hole is proportional to B. exist at the core of nearly all galaxies and that the mass of each black hole is proportional to that of C. exist at the core of nearly all galaxies, and that the mass of each black hole is proportional to D. exists at the core of nearly all galaxies, and that the mass of each black hole is proportional to that of E. exists at the core of nearly all galaxies and the mass of each black hole is proportional to that of

Open marker-平行结构难判断结束开始,平行短语句子会用同样signal word开头 Wrong: I want to retire to a place WHERE I can relax and i can pay low taxes Right: I want to retire to a place WHERE i can relax and Where I pay low taxes Correct Answer: B Selected Answer: A Logical predication; Agreement This sentence focuses attention on two hypotheses about black holes—one about their location and the other about their mass. These hypotheses appear as parallel relative clauses that black holes . . . exist at . . . and that the mass . . . is proportional. The subject of the first relative clause (black holes) is plural, so the clause must be completed with the plural form of exist. A This version of the sentence does not provide the relative pronoun that to provide parallel structure for presenting the two things scientific evidence reveals about black holes. This leaves it somewhat unclear whether the final clause is intended to convey part of what the scientists discovered or to express a claim that the writer is making independently of the scientists' discovery. The sentence illogically compares mass to galaxy. B Correct. The structure of the sentence makes the meaning clear, and the plural verb form agrees with the plural subject. C The comma appears to signal that the final part of the sentence is intended as an independent clause expressing a separate claim rather than describing part of the scientists' discovery. But if it were intended in that way, it should be set off as a separate sentence, not conjoined with the preceding clause. Like (A), this version makes an illogical comparison between mass and galaxy. D The singular verb exists does not agree with the plural subject black holes. As in (C), the comma is inappropriate because it does not introduce a new independent clause. E The singular verb exists does not agree with the plural subject black holes. Like (A), this version lacks the relative pronoun that, which would clarify the relationship between the two clauses.

like vs as vs such as

Part One: Comparisons First of all, the terms of comparisons must be in parallel, and the GMAT loves parallel structure. Furthermore, the comparisons on the GMAT are almost never (single word thing) vs. (single word thing), but, rather, nouns modified by extended phrases and clauses, so that one has to read carefully to sort out which two things are being compared. Finally, they love distinctions like the "like" vs. "as" distinction. *The word "like" is a preposition, whose object is a noun, so it's used for comparing noun-to-noun. The word "as" is a subordinating conjunction, which is followed by a full noun + verb clause, so it is used to compare events, actions.* -like as比较: like 比较名词, as 比较短语从句 1) Correct: Blue tits, like peacocks, demonstrate strong sexual dimorphism. 2) Incorrect: Blue tits, as peacocks, demonstrate strong sexual dimorphism. 3) Incorrect: Mahler died after composing his ninth symphony, like Beethoven and Dvorak had before him. 4) Correct: Mahler died after composing his ninth symphony, as Beethoven and Dvorak had before him. If the comparison is simply between nouns, use "like." If the comparison involves a full subject + verb clause, use "as." Part Two: Examples 举例-as举例要用such as, *like永远不能用来举例* The words "like" and "as" are also used in listing examples, but here we have to know a specific idiom. The unadorned "like" is used in colloquial speech, but the word "as" needs the word "such" — 5) Passerine birds like sparrows and swallows, ....WRONG 6) Passerine birds such as sparrows and swallows, ....RIGHT 7) Such passerine birds as sparrows and swallows, ....RIGHT Notice, the "such ... as" construction may involve the two words next to each other, or (as is far more likely on the GMAT SC!) separated by a word or words followed by an extended modifier. Notice also that the use of "like" for a list of examples is always incorrect on the GMAT. http://gmatclub.com/forum/as-vs-like-correct-and-incorrect-usages-133950.html LIKE AND AS -STATE COMPARISON Both 'like' and 'as' are used to state comparisons. For example: • Amy takes care of the children in the day care like a mother. • Amy takes care of the children in the day care as a mother does. Both sentences are correct. Both the sentences above convey the meaning that Amy takes care of the children "in the same way" as a mother takes care of her children. But notice the usage here - 'like' is followed by a noun and 'as' is followed by a clause. This grammatical construction should be kept in mind. AS - CAN ALSO PRESENT FUNCTION Apart from stating comparison, "as" can also be used to state "role" or "function". For example: John has joined the organization as a researcher. In this sentence, 'as' does not show a comparison. This sentence implies that John is indeed a researcher in the new organization. Notice that per this sentence John has not been compared to a researcher. John is indeed a researcher. So his role has been presented. And hence 'as' is used to present role or function of a person. Notice the grammatical construction - 'as' has been followed by a noun. This should always be kept in mind.

虚拟

The Subjunctive When we are talking about plain facts and truths we know for certain, we use the indicative mood. The subjunctive is for everything that's not so certain. We use the subjunctive to talk about: 1) Counterfactual possibilities — that is, hypothetical possibilities that, at the moment, simply are not true 2) Doubtful possibilities 3) Possibilities in the constructions such as "wish that", "desire that", or "lest that" The Subjunctive for Counterfactual Possibilities Even if something is not true at the moment, it may be an important possibility to consider. We use the past subjunctive in an "if" clause to discuss such counterfactual possibilities. If I were you, I would explain the situation to her immediately. Even if he had a million dollars, he still would complain about not having enough. If I read six books at once, I would confuse all the plots and characters. The past subjunctive uses the plural past tense form of the verb. For most verbs, the past tense is the same for singular & plural, but for the form "to be", the singular past tense ("was") is different from the plural past tense ("were"). This means, for most verbs, the past subjunctive will be indistinguishable from the past indicative; verb "to be" is the only verb that would reveal the different. Doubtful Possibilities Sometimes we need to plan for things that are unlikely, but that could happen. We use the future subjunctive in an "if" clause to discuss such possibilities. If I were to win the lottery, I finally would buy a new car. If I were to learn Sanskrit, I would understand etymology much better. If he were to be the next Picasso, I would be very happy to have known him for so long. Notice: the form of the future subjunctive is "were" + the infinitive of the verb. Wishes, Desires and "Lest" We use the present subjunctive to express any wish/desire/etc. that is stated in a clause beginning with "that." We also use the present subjunctive in clauses that follow the word "lest." My parents desire that I be famous. She hopes that she not be chosen for the committee. He wants to clear his name, lest he lose some of his civil rights. I study assiduously, lest I do poorly on the GMAT. These may sound awkward, because fewer than 1% of the population uses these grammatical forms correctly. Notice that the present subjunctive is merely the infinite form of the verb without the "to"; for many verbs, this is identical to the indicative present, but we don't add an "s" for the third-person singular (he/she/it).

The first detailed study of magpie attacks in Australia indicates that by the time they had reached adulthood, 98 percent of men and 75 percent of women born in country have been attacked by the birds. a) by the time they had reached adulthood, 98 percent of men and 75 percent of women born in country have been attacked by the birds. b) by the time they reach adulthood, 98 percent of men and 75 percent of women, who were born in the country, had been attacked by birds. c) by the time they reached adulthood, 98 percent of men and 75 percent of the women born in the country had been attacked by the birds. d) 98 percent of the men and 75 percent of the women that were born in the country were attacked by the birds by the time they reach adulthood. e) 98 percent of men and 75 percent of women who were born in the country, by the time they reached adulthood had been attacked by birds

The first detailed study of magpie attacks in Australia indicates that by the time they had reached adulthood, 98 percent of men and 75 percent of women born in the country have been attacked by the birds. It is very important to understand the intended meaning of the sentence to ascertain the timeline of the events. The first detailed study of magpie attacks in Australia suggests that by the time 98% of men and 75% of women reached adulthood, they had already been attacked by the bird. Analysis Notice that the sentence says that men and women had already been attacked by the bird even before they reached adulthood. This means that they got attacked first and then they reached adulthood. Now this can be reported in two ways: a. "by the time they reach adulthood, 98% of men and 75% of women have been attacked by the birds." (Reported in Present context) b. "by the time they reached adulthood, 98% of men and 75% of women had been attacked by the birds." (Reported in Past context) Error Analysis: 1. This sentence uses past perfect tense for the event that took place later and present perfect tense for the event that took place earlier. In order to establish the correct sequencing, we must use past perfect for the event that took place earlier, that is men and women being attacked, and simple past tense for the event that took place later, that is they reaching adulthood. POE: Choice A: by the time they had reached adulthood, 98 percent of men and 75 percent of women born in the country have been attacked by the birds. Incorrect for the verb tense error discussed above. Choice B: by the time they reach adulthood, 98 percent of men and 75 percent of women, who were born in the country, had been attacked by the birds. Incorrect. This choice repeats the verb tense error by using simple present tense for the event that took place later when past perfect has been correctly used for the earlier event. Choice C: by the time they reached adulthood, 98 percent of men and 75 percent of women born in the country had been attacked by the birds. Correct. The verb tense error has been corrected here. Choice D: 98 percent of men and 75 percent of women that were born in the country were attacked by the birds by the time they reach adulthood. Incorrect. a. "that" cannot be used to refer to "men and women". b. "reach" is not written in the correct verb tense. Choice E: 98 percent of men and 75 percent of women who were born in the country, by the time they reached adulthood had been attacked by the birds. Incorrect. We Have modifier ambiguity in this choice. Notice that "by the time they reached adulthood" is placed little awkwardly that makes it ambiguous that which entity it's modifying. It seems as if it is modifying "who were born in the country". This modifier should have a comma after adulthood to do away with this ambiguity. PS - Pronoun "they" in Choices A, B, C and E refers to "men and women" because only these men and women reached adulthood. There is no other antecedent "they" can logically refer to. Take away 1. Use past perfect for the event that took place earlier and simple past for the event that took place later. 2. Use the correct combination of verb tenses depending upon the context of the sentence. 3. There should not be any ambiguity in modifier reference.

The single family house constructed by the Yana, a Native American people who lived in what is now northern California, was conical in shape, its framework of poles overlaid with slabs of bark, either cedar or pine, and banked with dirt to a height of three to four feet. A) banked with dirt to a height of B) banked with dirt as high as that of C) banked them with dirt to a height of D) was banked with dirt as high as E) was banked with dirt as high as that of

The single family house constructed by the Yana (...) was conical in shape, its framework of poles overlaid with slabs of bark and banked with dirt to a height of three to four feet. A) "banked" is parallel with "overlaid" as under banked with dirt (to a height) of x feet overlaid with slabs of bark Hope that makes sense. B) "as high as" is unnecessary and "that of.." is inapproperiate. C) "them"? D) "was" is not paralalled with "overlaid" and is in passive. "as high as" is unnecessary for no comparision. E) Similar to D. "was" is not paralllel with "overlaid" and is in passive. "as high as" is unnecessary for no comparision. "that of"? _________________

Wind resistance created by opening windows while driving results in a fuel penalty as great or greater than is incurred by using air conditioning. (A) as great or greater than is incurred by using air conditioning (B) that is as great or greater than is incurred using air conditioning (C) as great as or greater than that of using air conditioning (D) at least as great as air conditioning's (E) at least as great as that incurred by using air conditioning

as great or greater ->redundancy A,B,C->out D->incorrect comparision E 开窗子自己不会有fuel penalty, fuel penalty incurred by opening window Wind resistance created by opening windows while driving results in a fuel penalty "as great as" that [wind resistance] incurred BY USING a/c

36. (Book Question: 805)Although she was considered among her contemporaries to be the better poet than her husband, later Elizabeth Barrett Browning was overshadowed by his success. A. Although she was considered among her contemporaries to be the better poet than her husband, later Elizabeth Barrett Browning was overshadowed by his success. B. Although Elizabeth Barrett Browning was considered among her contemporaries as a better poet than her husband, she was later overshadowed by his success. C. Later overshadowed by the success of her husband, Elizabeth Barrett Browning's poetry had been considered among her contemporaries to be better than that of her husband. D. Although Elizabeth Barrett Browning's success was later overshadowed by that of her husband, among her contemporaries she was considered the better poet. E. Elizabeth Barrett Browning's poetry was considered among her contemporaries as better than her husband, but her success was later overshadowed by his.

consider用法, Correct Answer: D Selected Answer: D Idiom; Verb form; Logical predication The sentence misuses the locution the better poet. It is acceptable to say a better poet than but not the better poet than. If you have already mentioned two poets X and Y, and you want to say that X is better than Y, you can either say X is a better poet than Y, or simply, X is the better poet. A This version of the sentence misuses the idiom the better poet, most likely confusing it with the idiom a better poet than. B [W]as considered among her contemporaries as is awkward and unnecessarily creates a potential misreading (she was considered to be among her contemporaries). C The subject here is Elizabeth Barrett Browning's poetry, not Elizabeth Barrett Browning, so the antecedent of her in her husband could comically be taken to be Elizabeth Barrett Browning's poetry. Furthermore, the tense of had been considered pairs oddly with Later overshadowed. Each seems to be in reaction to some particular time T—later than T and earlier than T, but when T was or its significance is never indicated. D Correct. This version avoids the problems of the other versions. E This sentence comically compares Elizabeth Barrett Browning's poetry to her husband.

56. (Book Question: 784)Although appearing less appetizing than most of their round and red supermarket cousins, heirloom tomatoes, grown from seeds saved during the previous year—they are often green and striped, or have plenty of bumps and bruises—heirlooms are more flavorful and thus in increasing demand. A. Although appearing less appetizing than most of their round and red supermarket cousins, heirloom tomatoes, grown from seeds saved during the previous year B. Although heirloom tomatoes, grown from seeds saved during the previous year, appear less appetizing than most of their round and red supermarket cousins C. Although they appear less appetizing than most of their round and red supermarket cousins, heirloom tomatoes, grown from seeds saved during the previous year D. Grown from seeds saved during the previous year, heirloom tomatoes appear less appetizing than most of their round and red supermarket cousins E. Heirloom tomatoes, grown from seeds saved during the previous year, although they appear less appetizing than most of their round and red supermarket cousins

dash可以表示强调,链接多个句子,每个句子完整 http://gmatclub.com/forum/although-appearing-less-appetizing-than-most-of-their-round-132487.html?kudos=1 Correct Answer: B Selected Answer: B Rhetorical construction; Grammatical construction The intended meaning could be communicated more effectively by mentioning heirloom tomatoes as early as possible in the sentence, so that we know that the writer is comparing heirloom tomatoes with supermarket tomatoes. The placement of heirloom tomatoes and heirlooms makes the sentence ungrammatical. A This is ungrammatical. If heirloom tomatoes is the subject of are more flavorful . . . then heirlooms has no predicate and is nonsensically superfluous. If heirlooms is the subject, heirloom tomatoes has no predicate. B Correct. The noun heirloom tomatoes is mentioned early in the sentence, followed by a parenthetical definition, and is the subject of the verb appear, and heirlooms is the subject of are. C The noun heirloom tomatoes appears too late in the sentence. Parsing is made harder by introducing the pronoun they and revealing its antecedent later in the sentence. The sentence is also ungrammatical. If heirloom tomatoes is the subject of are more flavorful . . . then heirlooms has no predicate and is nonsensically superfluous. If heirlooms is the subject, heirloom tomatoes has no predicate. D Beginning the sentence with the explanatory clause grown from seeds . . . gives it too much importance. It could be construed as the reason why heirloom tomatoes appear less appetizing, which is contrary to the truth. The sentence is also ungrammatical. E Rhetorical structure requires that although appear in the beginning of the clause to which it pertains. Placing it later necessitates the pronoun they with antecedent heirloom tomatoes, which is redundant. The sentence is also ungrammatical.

Most efforts to combat such mosquito-borne diseases like malaria and dengue have focused either on the vaccination of humans or on exterminating mosquitoes with pesticides. A. like malaria and dengue have focused either on the vaccination of humans or on exterminating B. like malaria and dengue have focused either on vaccinating of humans or on the extermination of C. as malaria and dengue have focused on either vaccinating humans or on exterminating D. as malaria and dengue have focused on either vaccinating of humans or on extermination of E. as malaria and dengue have focused on either vaccinating humans or exterminating

either or并列结构怎么用 Correct Answer: E Selected Answer: D Diction; Parallelism The phrase such . . . diseases like malaria and dengue is not a correct way in English to indicate that the two diseases mentioned are examples of a larger category; the correct expression is such . . . as. . . . A This use of such . . . like . . . is incorrect English; the correct expression is such . . . as.. . . It is better to keep the preposition on close to the verb it goes with, focus, so as not to repeat it. B The correct expression is such . . . as . . . . It is better to keep the preposition on close to the verb it goes with, focus, so as not to repeat it. This use of the gerund vaccinating (followed by of) would normally be preceded by the, but this would make the phrase awkward. It would be preferable to use vaccination, which is parallel to extermination. C This answer choice incorrectly repeats the preposition on before exterminating. D This answer choice incorrectly repeats the preposition on before extermination. This use of the gerund vaccinating (followed by of) would normally be preceded by the, but this would make the phrase awkward. It would be preferable to use vaccination, which is parallel to extermination. E Correct. This version uses either . . . or . . . correctly and appropriately uses the parallel forms vaccinating and exterminating.

34. (Book Question: 783)Mauritius was a British colony for almost 200 years, excepting for the domains of administration and teaching, the English language was never really spoken on the island. A. excepting for B. except in C. but except in D. but excepting for E. with the exception of

except是介词,要连接句子的话要用连词,but Correct Answer: C Selected Answer: E Idiom; Grammatical construction This two-clause sentence describes an apparent incompatibility: as a British colony, Mauritius might be expected to be English-speaking, but in fact it was not. To describe this apparent contradiction and to avoid a comma splice, the clauses should be joined by the conjunction but. Domains describes places in which English is spoken; for is the incorrect preposition. Excepting is not idiomatic English in this case. A The lack of a conjunction causes a comma splice; excepting for is non-idiomatic. B The lack of a conjunction causes a comma splice. C Correct. The two independent clauses are separated by but, and except in is an appropriate idiom. D Excepting for is non-idiomatic. E The lack of a conjunction causes a comma splice. The sentence includes two independent clauses, so a conjuntion should be used to connect two clauses. A, B, E out "excepting" in D modifies "the English language" --> awkward C is the winner

Since February, the Federal Reserve has raised its short-term interest rate target five times, and because of the economy's continued strength, analysts have been predicting for weeks that the target will be raised again in November. A. because of the economy's continued strength, analysts have been predicting for weeks that the target will B. with the economy's strength continuing, analysts predicted for weeks that the target C. because the economy continues strong, analysts predicted for weeks that the target would D. due to the economy's continued strength, analysts have been predicting for weeks that the target E. due to the fact of the economy's continued strength, analysts predicted for weeks that the target will

first of all we can see that in non underlined portion we can see a COMMA before and...hence a independent clause must be there after and. now option B is straight away wrong ..since an IC you will never see starting with WITH...HENCE B out. C. because the economy continues strong, analysts predicted for weeks that the target would===>PREDICTITION is not 100 percent THING...and so also WOULD...so never use WOULD with PREDICTION....its kinda redundant thing...always use WILL.==>Hence this is wrong D. due to the economy's continued strength, analysts have been predicting for weeks that the target===>absence of WILL makes this option wrong...its acting as a subjunctive form.==>hence wrong. E. due to the fact of the economy's continued strength, analysts predicted for weeks that the target will==>here they are 2 errors...first,due to the fact adds to wordiness .second, predicted ...is wrongverb tense ...we need present perfect ..because of the use of FOR WEEKS. Hence correct option is A. Three things always take "will" - prediction estimation calculation predictions should take the future tense, because they're predictions about ... the future!

83. (Book Question: 252)Many kitchens today are equipped with high-speed electrical gadgets, such as blenders and food processors, which are able to inflict as serious injuries as those caused by an industrial wood-planing machine. A. which are able to inflict as serious injuries as those B. which can inflict serious injuries such as those C. inflicting injuries as serious as that having been D. capable to inflict injuries as serious as that E. capable of inflicting injuries as serious as those

having been直接错 This is a comparison between injuries caused by kitchen gadgets and wood planning machines. A says that the gadgets can cause those serious injuries as caused by the planning machines. Whereas E says that, the gadgets can cause many injuries including as the serious ones caused by the machines. The logical follow up is to ask whether the gadgets cannot cause less serious injuries. Yes, they can and, therefore, E wins by practical rationale. Capable of or able to is just a ploy, as I see it. Correct Answer: E Selected Answer: B Idiom; Agreement The point of this sentence is the claim that common kitchen appliances can be as dangerous as an industrial wood-planing machine. It makes this point by comparing the injuries (plural) caused by blenders and food processors with those (also plural) caused by the wood-planing machine. An efficient way to make this comparison is to use the idiom capable of, an adjective phrase rather than a relative clause, after blenders and food processors. A The term able suggests agency, which kitchen gadgets do not have. The phrase as serious injuries as those is non-idiomatic, apparently *comparing injuries rather than the degree of seriousness of injuries. The relative clause makes the sentence unnecessarily wordy.*比较的平行问题 *B Like (A), this sentence introduces wordiness with a relative clause; the comparative phrase such as nonsensically suggests that injuries caused by a shop machine are examples of those caused by kitchen gadgets.* C It is not clear what the participial inflicting modifies. The sentence suggests that kitchen gadgets inevitably inflict injuries; the singular relative pronoun that either incorrectly refers to a plural noun, injuries, or implausibly indicates that industrial wood-planing machines have only ever caused a single injury. D As in (C), the singular pronoun that either disagrees with the plural noun injuries or implausibly indicates that industrial wood-planing machines have only ever caused a single injury. The phrase capable to is not idiomatic. E Correct. This version of the sentence correctly compares the seriousness of one type of injury with the seriousness of another (as serious as those), and the phrase capable of is a correct idiom.

关于分词修饰语的理解 - by tigercaiqun

http://gmatclub.com/forum/verb-ed-modifiers-vs-verb-ing-modifiers-125611.html Verb-ing modifiers 1: When separated by comma modifies the preceding clause 2: When not separated by comma modifiers the preceding noun or noun phrase Verb-ed Modifiers 1: Always modify preceding noun or noun phrase. verb-ed modifier is a noun modifier. When placed in the beginning of the sentence followed by a comma, it always modifies the subject of the clause. In case of the verb-ing modifiers, when places before the clause separated by a comma they can modify either the subject or the entire clause, depending upon the context of the sentence. 1.当过去分词在句末时,不管有无逗号,都是就近修饰名词; 2.现在分词在句末时,无逗号是就近修饰的名词的定语; 3.当现在分词在句尾且前有逗号时,修饰邻近句子的主语,做主语的伴随状语或做句子的伴随结果.因此不能用句尾现在分词修饰前面句尾的名词. 所以OG127说C选项:"the phrase having been assigned...is uncertain in reference,making the sentence unclear."-->就是说这个现在分词本来是应该修饰前面的employee的,却变成了修饰主语governments了. (另外在GMAT里,having been done的用法错误,应直接用done) 避免上述错误的方法:所修饰的名词在句尾用定语从句修饰--OG127正确选项B (而不用加逗号的现在分词修饰.其实分词是定语从句的省略形式,在句中无逗号分词(注意是无逗号的,有逗号的在句中有歧义)就比定语从句简洁)还有 OG120也是同样道理:修饰句尾名词,正确选项A用定语从句.E选项的句尾现在分词错误. 4.当现在/过去分词在句中且前没有逗号,修饰前面紧邻名词; 5.当现在/过去分词在句中且前后都有逗号,有歧义:1)修饰前面紧邻的名词,2)向后修饰后面句子的主语. 这种结构在GMAT肯定错,如果修饰某句主语,则避免将该分词置于以名词结尾的句后.避免方式:1)用定语从句/介词短语明确修饰对象.2)可将分词提到句首,所修饰主语及所在句子紧跟其后-->形成句首分词修饰句子主语.见OG179 6.在前面有多个名词如名词1+介词+名词2结构,而要用分词修饰名词1时,为避免歧义要重复名词1即用同位语结构:名词1+介词+名词2,名词1+分词.见OG208。

103. (Book Question: 777)According to recent studies comparing the nutritional value of meat from wild animals and meat from domesticated animals, wild animals have less total fat than do livestock fed on grain and more of a kind of fat they think is good for cardiac health. A. wild animals have less total fat than do livestock fed on grain and more of a kind of fat they think is B. wild animals have less total fat than livestock fed on grain and more of a kind of fat thought to be C. wild animals have less total fat than that of livestock fed on grain and have more fat of a kind thought to be D. total fat of wild animals is less than livestock fed on grain and they have more fat of a kind thought to be E. total fat is less in wild animals than that of livestock fed on grain and more of their fat is of a kind they think is

http://www.beatthegmat.com/according-to-recent-studies-comparing-the-nutritional-t84907.html 这道题有一个语法点那就是 如果比较前后两个动词一样,那么后面的那个动词是可以被省略的。 比如B:wild animals (have) less total fat than livestock fed on grain (have/do) and more of a kind of fat thought to be 后面的那个have是补出来的。因为这个have可以被省略,A就变成了不好的选项(其实A也是对的) 还有几道题也存在这个语法点 Salt deposits and moisture threaten to destroy the Mohenjo-Daro excavation in Pakistan, the site of an ancient civilization that flourished at the same time as the civilizations in the Nile delta and the river valleys of the Tigris and Euphrates. (A) that flourished at the same time as the civilizations (B) that had flourished at the same time as had the civilizations (C) that flourished at the same time those had (D) flourishing at the same time as those did (E) flourishing at the same time as those were 这道题A选项: that (flourished) at the same time as the civilizations (flourished/did) 还有一道题 Traffic safety officials predict that drivers will be equally likely to exceed the proposed speed limit as the current one. A. equally likely to exceed the proposed speed limit as B. equally likely to exceed the proposed speed limit as they are C. equally likely that they will exceed the proposed speed limit as D. as likely that they will exceed the proposed speed limit as E. as likely to exceed the proposed speed limit as they are. 这道题E选项as likely (to exceed) the proposed speed limit as they are (to exceed) Logical predication; Rhetorical construction The sentence reports research findings on the comparison between the fat content of wild animals and that of domestic livestock. The most significant error in the sentence is in the phrase they think: the pronoun they either lacks a referent or is meant to refer back to wild animals, which would be nonsensical. A The pronoun they fails to refer correctly. B Correct. The phrase thought to be eliminates the most significant error in the original sentence. Note that while the phrase less total fat than livestock differs from the phrase less total fat than do livestock in the original, either would be correct here. C The resulting sentence is unnecessarily wordy and confusing. The pronoun that is not only superfluous, but it fails to refer back to anything. D The resulting sentence makes a nonsensical comparison between total fat and livestock. E The resulting sentence is wordy and confusing. It lacks the required parallelism in wild animals . . . in livestock. The referent of the possessive pronoun their is ambiguous, as is the referent of the pronoun they.

42. (Book Question: 679)In 1979 lack of rain reduced India's rice production to about 41 million tons, nearly 25 percent less than those of the 1978 harvest. A. less than those of the 1978 harvest B. less than the 1978 harvest C. less than 1978 D. fewer than 1978 E. fewer than that of India's 1978 harvest

less不可数,fewer可数 Correct Answer: B Selected Answer: B Logical predication; Diction The crucial part of this sentence is the latter part, where India's 1979 production of 41 million tons of rice is compared to the production of the previous year. What was produced in 1979 was 25 percent less than what was produced in 1978. The grammatical means of comparison have to express correctly this relationship between the two harvests. It helps if you think of 41 million tons of rice as the collective equivalent of the 1979 harvest. A This version exhibits redundant word choice. The pronoun those refers to the tons of the 1978 harvest. Both harvests are measured in tons, and it is clearer and simpler to compare with the harvest itself, not with the tons of the harvest. B Correct. This version correctly compares the 1978 harvest with the 1979 harvest. C This version is illogical. It compares rice production in tons with a year, 1978. D In addition to the problem described in (C) above, this version uses the wrong comparative adjective. Fewer refers to countable nouns, while less refers to quantities. Since the comparative adjective should refer to rice production, fewer is not appropriate. E Since the comparative adjective should refer to rice production, fewer is not appropriate. Furthermore, the pronoun that refers back to India's rice production [fewer than the rice production of India's 1978 harvest] and is redundant. The word harvest is sufficient here to express the comparison.

54. (Book Question: 761)Recently physicians have determined that stomach ulcers are not caused by stress, alcohol, or rich foods, but a bacterium that dwells in the mucous lining of the stomach. A. not caused by stress, alcohol, or rich foods, but B. not caused by stress, alcohol, or rich foods, but are by C. caused not by stress, alcohol, or rich foods, but by D. caused not by stress, alcohol, and rich foods, but E. caused not by stress, alcohol, and rich foods, but are by

not by but by不用两个be Correct Answer: C Selected Answer: B Parallelism; Diction The formula used in this sentence not this but that requires parallel elements following not and but. This means that not by stress, alcohol, or rich foods must be balanced by but by a bacterium. . . . There is no need to repeat the verb are caused, or even the auxiliary verb are, because the verb precedes the not by . . . but by . . . formula. The substitution of the conjunction and for the conjunction or changes the meaning of the sentence: Stress, alcohol and rich foods identifies the combination of these three factors as a suggested cause of stomach ulcers, whereas stress, alcohol, or rich foods offers three individual possibilities. There is no way to tell which one of these is the intended meaning of the sentence. A To preserve parallelism, but should be followed by by. B There is no reason to repeat the auxiliary verb are. C Correct. This sentence correctly uses the not by . . . but by . . . formula. D To preserve parallelism, but should be followed by by. E To preserve parallelism, but should be followed by by.

Guide-8, SC, Chapter-5, page-73, Q6 改错 Caroline receives e-mail from friends who she knows well, from acquaintances who's names are only vaguely familiar, and from strangers about who she knows nothing at all.

object pronoun-who subject pronoun-whom possessive pronoun-whose pronouns antecedents Answer: Caroline receives e-mail from friends whom she knows well, from acquaintances whose names are only vaguely familiar, and from strangers about whom she knows nothing at all.

句子改错-Helen would feel better if she was my daughter

was- past tense of indicative mood, should be were-present tense of hypothetical subjunctive mood Helen would feel better if she were my daughter

18. (Book Question: 775)The stars, some of them at tremendous speeds, are in motion just as the planets are, yet being so far away from Earth that their apparent positions in the sky do not change enough for their movement to be observed during a single human lifetime. A. The stars, some of them at tremendous speeds, are in motion just as the planets are, yet being B. Like the planets, the stars are in motion, some of them at tremendous speeds, but they are C. Although like the planets the stars are in motion, some of them at tremendous speeds, yet D. As the planets, the stars are in motion, some of them at tremendous speeds, but they are E. The stars are in motion like the planets, some of which at tremendous speeds are in motion but

yet being so far away 不是一个主语,没有动词, c选项中错误在于用as来作比较,是必然错误的 Correct Answer: B Selected Answer: E Grammatical construction; Rhetorical construction The first part of the original sentence intends to compare stars and planets; the comparison would be more effective at the beginning of the sentence: Like the planets, the stars. This alternative construction would lead the reader to expect the verb are immediately following the subject, and then the completion of the clause, in motion. The modifying phrase, some of them at tremendous speeds, is best placed after motion. This whole construction, Like the planets, the stars are in motion, some of them at tremendous speeds, is a main clause and must be followed by a comma before a coordinating conjunction (such as yet or but) introduces a second main clause. The second clause must have a subject and a verb; being is neither and must be replaced with they are. A Placements of the modifying phrase and the comparison are awkward and ineffective; being provides neither a subject nor a verb for the second main clause. B Correct. The comparison is clear and effective in this sentence; the second clause includes a subject and a verb. C Both although and yet indicate contrast, so only one of them may be used; wordy, awkward phrasing leads to an ungrammatical construction that lacks a subject and verb for the second clause. D The preposition like must be used for a comparison of two nouns; the subordinating conjunction as would need to introduce a subordinate clause. E Placement of like the planets is awkward; some of which is awkward and ambiguous; are in motion is said twice; subject and verb of the second clause are omitted.

110. (Book Question: 788)Among the objects found in the excavated temple were small terra-cotta effigies left by supplicants who were either asking the goddess Bona Dea's aid in healing physical and mental ills or thanking her for such help. A. in healing physical and mental ills or thanking her for such help B. in healing physical and mental ills and to thank her for helping C. in healing physical and mental ills, and thanking her for helping D. to heal physical and mental ills or to thank her for such help E. to heal physical and mental ills or thanking her for such help

不要忘记了either or是怎么用的 Correct Answer: A Selected Answer: D Parallelism; Idiom This correct sentence uses parallel structure to explain that supplicants were either asking . . . or thanking. The correlative pair either/or is correctly used since each element is followed by the same part of speech: either asking . . . or thanking. The pair of correlative conjunctions either . . . or always work together; either may only be followed by or. The noun aid is correctly followed by in healing rather than by the infinitive to heal. A Correct. The original sentence uses parallel structure to make its point; the idioms are correctly used. B And is incorrect following either, and its use changes the meaning of the sentence; to thank is not parallel to asking; for helping is awkward. C No comma should be used following ills; and is incorrect following either, and its use changes the meaning of the sentence; for helping is awkward. D To heal is incorrect following aid; to thank is not parallel to asking. E To heal is incorrect following aid.

40. (Book Question: 684)Like ancient Egyptian architectural materials that were recycled in the construction of ancient Greek Alexandria, so ancient Greek materials from the construction of that city were reused in subsequent centuries by Roman, Muslim, and modern builders. A. Like ancient Egyptian architectural materials that were recycled in the construction of B. Like recycling ancient Egyptian architectural materials to construct C. Just as ancient Egyptian architectural materials were recycled in the construction of D. Just as they recycled ancient Egyptian architectural materials in constructing E. Just like ancient Egyptian architectural materials that were recycled in constructing

举例的时候不能用like,一定要用as Correct Answer: C Selected Answer: E Diction; Parallelism When two situations are asserted to be similar, the proper way to express this is with the paired expressions just as . . . so . . ., not like . . . so. Moreover, the two compared situations should be expressed as clauses, not as noun phrases. Thus the clause ancient Egyptian architectural materials were recycled . . . is correct, as opposed to a noun phrase like ancient Egyptian architectural materials that were recycled . . . A Just as and a following clause with a passive verb are preferred, but instead this option has like and a following noun phrase (ancient Egyptian materials . . .). It appears, somewhat implausibly, to say that the ancient Greek materials were similar to the earlier ancient Egyptian ones in that both were used by Roman, Muslim, and modern builders. B Just as and a following clause with a passive verb are preferred, but instead this option has like and a following noun phrase (recycling ancient Egyptian materials . . .). It appears illogically to say that the action of recycling was similar to the ancient Greek materials in that both were reused in subsequent centuries. C Correct. The expressions just as and so are paired to link the two clauses in parallel, and both clauses use the passive construction. D Just as is used to connect two clauses, which is good, but the first clause employs the active construction rather than the preferred passive, so there is a failure of parallelism. Also, it is unclear what they refers to. The sentence appears to say illogically that some unidentified group's action of recycling was similar to the ancient Greek materials' being reused. E Just as introducing a clause with a passive verb is preferable, but this sentence uses like and a following noun phrase (ancient Egyptian materials . . .). It appears, somewhat implausibly, to say that the ancient Greek materials were similar to the earlier ancient Egyptian ones in that both were used by Roman, Muslim, and modern builders.

139. (Book Question: 792)She was less successful after she had emigrated to New York compared to her native Germany, photographer Lotte Jacobi nevertheless earned a small group of discerning admirers, and her photographs were eventually exhibited in prestigious galleries across the United States. A. She was less successful after she had emigrated to New York compared to B. Being less successful after she had emigrated to New York as compared to C. Less successful after she emigrated to New York than she had been in D. Although she was less successful after emigrating to New York when compared to E. She had been less successful after emigrating to New York than in

出了比较时态 less这种就一定要有than Correct Answer: C Selected Answer: B Idiom; Grammatical construction; Logical predication This sentence compares the success Jacobi experienced after moving to New York to the success she had previously experienced in Germany. The phrase less successful anticipates the conclusion of the comparison with the phrase than. . . . The main subject of the sentence is photographer Lotte Jacobi, and the main verb is earned. The opening clause She was less successful . . . therefore creates a comma splice if the comma is not followed by a conjunction. The most efficient way to incorporate the information about Jacobi's comparative successes in Germany and in New York is to turn this clause into an adjectival phrase describing Jacobi. A Less successful . . . anticipates than rather than compared to . . .; a comma is insufficient to join two independent clauses into a single sentence. B As compared to is an incorrect way to complete the comparison introduced by less; Being . . . is unnecessarily wordy and awkward. C Correct. The idiomatic construction less successful . . . than is incorporated into an introductory adjectival phrase modifying Lotte Jacobi. D When compared to is an incorrect phrase to complete the comparison introduced by less. E A comma is insufficient to join two independent clauses into a single sentence; past-perfect tense is misleading, since it refers to Jacobi's experience in New York, which in fact followed her experience in Germany.

分词短语逻辑主语的判断

分词短语逻辑主语的判断 a) 分词短语在句首作状语:逻辑主语=主句主语 b) ing分词短语在句尾: 表伴随动作/状态/功能,与句子谓语动作同时发生,逻辑主语=句子主语 表伴随结果,整个句子是原因,导致分词动作产生,无逻辑主语.可以在分词前加thus/thereby/in effect/in fact等,也可以不加 (现在分词做状语表结果,OG154和259。) c) ed分词短语在句尾,一般优先就近作定语,修饰名词[OG127] d) ing分词和ed分词在句中:优先作定语修饰就近的名词[OG191] e) 介词或介词短语+ing分词: in addition to/in/by/without/besides+doing...,句子 在句首,逻辑主语=句子主语 在句尾,逻辑主语可能是句子主语也可能不是

24. (Book Question: 739)In some types of pine tree, a thick layer of needles protects the buds from which new growth proceeds; consequently they are able to withstand forest fires relatively well. A. a thick layer of needles protects the buds from which new growth proceeds; consequently they are able to withstand forest fires relatively well B. a thick needle layer protects buds from where new growth proceeds, so that they can withstand forest fires relatively well C. a thick layer of needles protect the buds from which new growth proceeds; thus, they are able to withstand relatively well any forest fires D. since the buds from which new growth proceeds are protected by a thick needle layer, consequently they can therefore withstand forest fires relatively well E. because the buds where new growth happens are protected by a thick layer of needles, they are able to withstand forest fires relatively easily as a result

副词修饰的位置 Correct Answer: A Selected Answer: C Grammatical construction; Rhetorical construction This sentence is fine as written. It uses the correct from which to introduce the relative clause modifying buds and avoids redundant expressions of causation, such as consequently . . . therefore, or because . . . as a result. A Correct. The relative clause starting with from which is in the correct form, and the causality is expressed efficiently and clearly with one word, consequently. B In this context, needle layer is less precise than the more standard layer of needles, which makes it clear that the layer is composed of needles rather than being, for example, a layer of a needle. From where is not the correct form, because it is redundant in using two words that express the idea of location (from and where) instead of one. C The short direct object any forest fires is separated from its verb withstand by an adverb phrase; this word order is awkward, and is acceptable only with very long direct objects and in some cases where there is no other reasonable way to eliminate ambiguity. D This version is unnecessarily redundant in expressing causation, using all of since, consequently, and therefore. As in answer choice B, layer of needles would be more precise than needle layer. E This version is unnecessarily redundant in expressing causation, using both because and as a result. 1st split from which vs from where -----> from where is used from places and from which is used for items or things . from which is correct . C a thick layer (singular) thus protects is needed . A vs D D is passive . Also "therefore" used is redundant when since/because is used . Thus A is answer .

独立主格和同位语

四种修饰包括:定语、同位语、状语(独立主格属于特例之一)和插入语。 同位语修饰名词或者短语,必须对修饰对象加油解释力,在修饰名词前后。结构:1)n, n; 2)n, a/an +n; 3) a/an n, n; 4) the+n, n;5)n+that clause; 6) n, n(重复修饰名词)+that clause; 7)n, one/ones+that clause OG 13 sc104 Yellow jackets number among the 900 or so species of the world's social wasps, wasps living in a highly cooperative and organized society where they consist almost entirely of females-the queen and her sterile female workers. (A) wasps living in a highly cooperative and organized society where they consist almost entirely of (B) wasps that live in a highly cooperative and organized society consisting almost entirely of (C) which means they live in a highly cooperative and organized societY, almost all (D) which means that their society is highly cooperative, organized, and it is almost entirely (E) living in a society that is highly cooperative, organized, and it consists of almost all 答案B,wasps+that从句做同位语解释wasps社会的特点。 独立主格在句首或者句尾,做状语,表伴随原因、条件或状态。独立主格不跟从句,只有:1)n+n.; 2)n+_ed/_ing; 3)n+介词短语;4) n+形容词短语;或者以上结果前带with; 最后句尾each型主格,each带以上四种结构。 Verbal 2 sc 75 Scientists have observed large concentrations of heavy-metal deposits in the upper twenty centimeters of Baltic Sea sediments, which are consistent with the growth of industrial activity there. (A) Baltic Sea sediments, which are consistent with the growth of industrial activity there (B) Baltic Sea sediments, where the growth of industrial activity is consistent with these findings (C) Baltic Sea sediments, findings consistent with its growth of industrial activity (D) sediments from the Baltic Sea, findings consistent with the growth of industrial activity in the area (E) sediments from the Baltic Sea, consistent with the growth of industrial activity there 答案D, 独立主格finds sonsistent with是 n+adj.形式,表明伴随原因

Tropical bats play important roles in the rain forest ecosystem, aiding in the dispersal of cashew, date, and fig seeds; /pollinating banana, breadfruit, and mango trees; and indirectly help produce/ tequila by pollinating agave plants. A. pollinating banana, breadfruit, and mango trees; and indirectly help produce B. pollinating banana, breadfruit, and mango trees; and indirectly helping to produce C. pollinating banana, breadfruit, and mango trees; and they indirectly help to produce D. they pollinate banana, breadfruit, and mango trees; and indirectly help producing E. they pollinate banana, breadfruit, and mango trees; indirectly helping the producing of

平行,分号用法 Correct Answer: B Selected Answer: D Logical predication; Parallelism This sentence expresses a list of the roles tropical bats play in the rain forest ecosystem. Since these roles are enumerated in a list, and since the first member of the list is already provided, it is necessary to maintain the same structure for the rest of the members of the list in order to maintain parallelism and clarity. */Note that semicolons separate the members of the list, leaving the commas to mark series of items within each member of the list./* A In this version, the third member of the list does not maintain the -ing verb form that the two previous members use. B Correct. This version correctly maintains the parallel structure (aiding in . . .; pollinating . . .; and helping . . .). C In this version, the third member of the list does not maintain the -ing verb form of the two previous members of the list. In addition, this member of the list includes a subject (they) while the other members do not, again violating parallelism. D In order to maintain parallelism the verb that is the member of the list has to be in the -ing form, not its complement. Thus, the -ing has to be on the verb help, not on produce. E Although this version maintains parallelism throughout, the phrase helping the producing is an incorrect construction in English. 分号两种用法:链接句子,或者表示转折,这里这种表示连接两个clause 也可以 The semicolon is often followed by a transition expression, such as however, therefore, or in addition. In this way, the writer can modify the equal relationship that a bare semicolon implies. Note that these transitional elements are not true conjunctions like and. As a result, you must use semicolons, not commas, to join the sentences: Right: Andrew and Lisa are inseparable; THEREFORE, we never see them apart. A rare but correct use of the semicolon is to separate items that themselves contain commas: Wrong: I listen to Earth, Wind & Fire, Wow, Owls, and Blood, Sweat & Tears. Right: I listen to Earth, Wind & Fire; Wow, Owls; and Blood, Sweat & Tears.

句子改错-Helen may feel better if she would swallow this pill

改错-if would一起一定错,Helen may feel better if she swallows this pill If 用indicative mood,

111. (Book Question: 240)Reporting that one of its many problems had been the recent extended sales slump in women's apparel, the seven-store retailer said it would start a three-month liquidation sale in all of its stores. A. its many problems had been the recent B. its many problems has been the recently C. its many problems is the recently D. their many problems is the recent E. their many problems had been the recent

时态问题 Correct Answer: A Selected Answer: C Agreement; Verb form; Diction The correct use of pronoun reference, verb tense, and modifier make the sentence clear and easy to understand. The singular possessive pronoun its refers to the singular noun retailer. The past-perfect verb had been indicates action completed before the action in the simple past tense said. The adjective recent modifies extended sales slump. A Correct. Its agrees with retailer; the past perfect had been indicates action prior to the simple past said; and recent modifies extended sales slump. B The adverb recently modifies only the adjective extended, suggesting illogically that the sales slump has been recently extended. C Is shows present, rather than completed, action, and the adverb recently modifies only the adjective extended, distorting meaning. D Their does not agree with retailer, and is shows present, rather than completed, action. E The plural their does not agree with the singular retailer.

116. (Book Question: 721)Along the major rivers that traverse the deserts of northeast Africa, the Middle East, and northwest India, the combination of a reliable supply of water and good growing conditions both encouraged farming traditions that, in places, endure in at least 6,000 years. A. good growing conditions both encouraged farming traditions that, in places, endure in B. good growing conditions encouraged farming traditions that have, in places, endured for C. of good growing conditions have encouraged farming traditions that, in places, endured for D. of good growing conditions both encouraged farming traditions that have, in places, endured E. of good growing conditions encouraged farming traditions that have, in places, been enduring for

看起来supply of good condition, correct to phrase to use is "combination of X and Y" not combination of X and of Y". This leaves us with choice (A) and (B) Correct Answer: B Selected Answer: C Logical predication; Rhetorical construction The time line of this sentence, captured by the use of verb tenses, is of utmost importance. A combination of factors (in the past) encouraged farming traditions that are still with us today. The conditions for the use of the present perfect tense have endured are in place. A The word both repeats the meaning of combination and is thus redundant. The use of the present tense (endure) is not justified by the time line of the whole sentence. The correct preposition for this type of construction is for (an amount of time), not in. B Correct. This version correctly employs the present perfect tense with the appropriate adverbial for at least 6,000 years. C The repetition of the preposition of before good growing conditions makes no sense. It seems to indicate that there is both a combination of a reliable supply of water and a combination of good growing conditions. D The preposition of should not be repeated in front of good growing conditions. The word both repeats the meaning of combination and is thus redundant. E The preposition of should not be repeated in front of good growing conditions. The use of the present perfect progressive have been enduring is not grammatically incorrect, but it is rhetorically inappropriate and sounds exaggerated.

67. (Book Question: 684)Like ancient Egyptian architectural materials that were recycled in the construction of ancient Greek Alexandria, so ancient Greek materials from the construction of that city were reused in subsequent centuries by Roman, Muslim, and modern builders. A. Like ancient Egyptian architectural materials that were recycled in the construction of B. Like recycling ancient Egyptian architectural materials to construct C. Just as ancient Egyptian architectural materials were recycled in the construction of D. Just as they recycled ancient Egyptian architectural materials in constructing E. Just like ancient Egyptian architectural materials that were recycled in constructing

记住用法,just as....so Correct Answer: C Selected Answer: E Diction; Parallelism When two situations are asserted to be similar, the proper way to express this is with the paired expressions just as . . . so . . ., not like . . . so. Moreover, the two compared situations should be expressed as clauses, not as noun phrases. Thus the clause ancient Egyptian architectural materials were recycled . . . is correct, as opposed to a noun phrase like ancient Egyptian architectural materials that were recycled . . . A Just as and a following clause with a passive verb are preferred, but instead this option has like and a following noun phrase (ancient Egyptian materials . . .). It appears, somewhat implausibly, to say that the ancient Greek materials were similar to the earlier ancient Egyptian ones in that both were used by Roman, Muslim, and modern builders. B Just as and a following clause with a passive verb are preferred, but instead this option has like and a following noun phrase (recycling ancient Egyptian materials . . .). It appears illogically to say that the action of recycling was similar to the ancient Greek materials in that both were reused in subsequent centuries. C Correct. The expressions just as and so are paired to link the two clauses in parallel, and both clauses use the passive construction. D Just as is used to connect two clauses, which is good, but the first clause employs the active construction rather than the preferred passive, so there is a failure of parallelism. Also, it is unclear what they refers to. The sentence appears to say illogically that some unidentified group's action of recycling was similar to the ancient Greek materials' being reused. E Just as introducing a clause with a passive verb is preferable, but this sentence uses like and a following noun phrase (ancient Egyptian materials . . .). It appears, somewhat implausibly, to say that the ancient Greek materials were similar to the earlier ancient Egyptian ones in that both were used by Roman, Muslim, and modern builders.

110. (Book Question: 237)Inspired by the Helsinki Accords and outraged by the harsh sentences meted out to a group of Czech rock musicians called the Plastic People of the Universe, Charter 77 was established by dissident writers, philosophers, and other professionals to be a human rights group. A. Charter 77 was established by dissident writers, philosophers, and other professionals to be B. Charter 77 had been established by dissident writers, philosophers, and other professionals as C. Charter 77, established by dissident writers, philosophers, and other professionals, was D. dissident writers, philosophers, and other professionals established Charter 77 as E. dissident writers, philosophers, and other professionals had established Charter 77 to be

过去完成时一定要在有必要的时候采用 Correct Answer: D Selected Answer: A Logical predication; Verb form This sentence explains what inspired a group of people to establish a human rights group called Chapter 77. The passive construction in the main clause illogically makes the opening phrase (inspired . . . and outraged) describe Charter 77. The sentence says that Charter 77 was established by a group of people. Thus, Charter 77 refers in this context to the organization as an abstract entity, not to its founders or members. The intended meaning presumably is that the dissident writers, philosophers, and other professionals were both inspired and outraged and were thus prompted to start Charter 77. A The opening adjectival phrase inappropriately describes Charter 77 instead of the people who started it. The idiomatic expression established . . . as is preferable to the nonstandard and somewhat unclear established . . . to be. . . . B The past-perfect verb form had been established is confusing after the past tense phrase meted out to a group. . . . Additionally, like (A), this version of the sentence creates an opening that somewhat illogically modifies Charter 77. C As in (A) and (B), the opening phrase illogically describes Charter 77 instead of the people who were inspired and outraged and thus prompted to start Charter 77. D Correct. The opening phrase correctly describes the subject dissident writers, philosophers, and other professionals, and the active verb established prevents unnecessary wordiness. E Although the opening phrase describes the people who started Charter 77, the past-perfect tense of the main clause is confusing, making the temporal relationships among the events unclear. The idiomatic expression established . . . as is preferable to the nonstandard and somewhat unclear established . . . to be.

19. (Book Question: 724)At the end of 2001, motion picture industry representatives said that there were about a million copies of Hollywood movies available online and expected piracy to increase with high-speed Internet connections that become more widely available. A. online and expected piracy to increase with high-speed Internet connections that become more widely available B. online and expect the increase of piracy with the wider availability of high-speed Internet connections C. online, and they expect more piracy to increase with the wider availability of high-speed Internet connections D. online, and that they expected the increase of piracy as high-speed Internet connections would become more widely available E. online, and that they expected piracy to increase as high-speed Internet connections became more widely available

这里同时考了下open markers, 两个并列句子要都加上signal word, where, that这种 When open markers are used, it is not as easy to see where the X element begins, especially in longer sentences. Often, the two parallel phrases or clauses may begin with the same signal word in order to remove ambiguity about where the parallelism begins: Wrong: I want to retire to a place WHERE I can relax AND Lrgv.low taxes. Right: I want to retire to a place WHERE I can relax AND WHERE I rgv.low taxes. Correct Answer: E Selected Answer: E Rhetorical construction; Grammatical construction Every clause needs a subject, either an overt subject or an understood subject (whose interpretation can come from a coordinated clause or some other nearby clause). In this sentence, the clause containing expected lacks a clear subject. The intended subject is motion picture industry representatives, but to clearly indicate that, the subject should either be repeated or be replaced with the pronoun they. Furthermore, piracy to increase with high-speed Internet connections that become more widely available is awkward, and it fails to clearly communicate the idea that piracy will increase as a result of high-speed Internet connections becoming available. A The second clause is awkward and unclear; there is no clear subject for expected. B There is no clear subject for expect. C It is redundant to use both more and increase. D This wording makes the meaning very unclear. They expected the increase in piracy appears to refer to a particular (past) increase, but this does not clearly make sense with the ensuing use of the conditional verb form would become, which is inappropriate here. E Correct. In this version the verb expect has an overt subject, and the following phrasing clearly indicates that the expected increase in piracy is the result of high-speed Internet connections becoming more widely available.

20. (Book Question: 806)In no other historical sighting did Halley's Comet cause such a worldwide sensation as did its return in 1910-1911. A. did its return in 1910-1911 B. had its 1910-1911 return C. in its return of 1910-1911 D. its return of 1910-1911 did E. its return in 1910-1911

难的并列项目 Correct Answer: C Selected Answer: A Parallelism; Verb form; Logical predication The single subject of this sentence is Halley's Comet, and its single verb phrase is did cause. The comparison presented by the sentence is between adverbial phrases describing times when the comet was seen. Grammatically, the items being compared are parallel prepositional phrases beginning with the preposition in: in no other sighting and in its return in 1910-1911. This is the clearest, most economical way of presenting the information. The options that introduce a second verb (did or had) violate the parallelism and introduce a comparison between the comet itself (subject of the verb did cause) and the comet's return (subject of the verb did or had). A This sentence implies a comparison between the comet and its return. B This sentence implies a comparison between the comet and its return; had is the wrong auxiliary verb form because it must be followed by caused instead of cause. C Correct. The parallel prepositional phrases in this sentence correctly compare times when the comet was sighted. D This sentence implies a comparison between the comet and its return. E This sentence violates parallelism, implying a comparison between a prepositional phrase and a noun phrase.


Related study sets

QUINTO GRADO, ¿De qué color es la bandera?

View Set

Propositional Logic Definitions (Ch. 7)

View Set

Español 322 - Capítulo 10 - Posición de los Adjetivos

View Set

PrepU 39: Management of Patients With Oral and Esophageal Disorders

View Set

Chapter 1 Quiz - Introduction to Marketing - Vincennes University - Professor Murphy's Course

View Set

Maternity and Women's Health NGN Case Study

View Set